*NURSING > QUESTIONS & ANSWERS > ANCC IQ Domains 1-5 Test Bank answered all correctly answered; latest updated summer 2022. (All)

ANCC IQ Domains 1-5 Test Bank answered all correctly answered; latest updated summer 2022.

Document Content and Description Below

ANCC Domain 1: Scientific Foundation (40 questions with rationales) As a PMHNP, you are aware of antipsychotic medication side effects. Which of the following side effects of antipsychotic medicati... ons has an insidious, delayed-onset typically 1 to 2 years after initiation of the medication? a. Dry mouth b. Tardive dyskinesia c. Sedation d. Weight gain In evaluating a 4-year-old, one would expect which of the following delays in Asperger's Disorder? a. Language development b. Cognitive development c. Motor skills coordination d. Social skills development Which area of the brain has been implicated to modulate pain and can help explain how transcranial magnetic stimulation can help decrease pain? a. Prefrontal cortex b. Insula cortex c. Dorsolateral prefrontal cortex d. Anterior cingulate cortex Which of the following antidepressants is associated with dose-related cardiovascular adverse effects which require adherence to maximum recommended dosing, unless higher doses are deemed appropriate despite risks? a. Citalopram (Celexa) b. Venlafaxine (Effexor) c. Mirtazapine (Remeron) d. Sertraline (Zoloft) Which of the following findings would raise concern in an annual sports exam for a 15-year-old girl? a. Scar tissue from hymenal tear at 11 o'clock (reported biking injury) b. Blood pressure 86/56 c. Height 67 inches, weight 102 pounds d. Pulse 60, respirations 26 A week after raising the dose of risperidone (Risperdal), a patient treated for schizophrenia presents to the clinic with reports of an acute change in mental status, fever, and rigidity. As the treating PMHNP, you know these symptoms are consistent with which of the following? a. Neuroleptic malignant syndrome (NMS) b. Anticholinergic withdrawal syndrome c. Extrapyramidal side effects d. Serotonin syndrome Which of the following statements regarding disulfiram (Antabuse is correct? a. Disulfiram should not be taken for at least 6 hours after drinking alcohol. b. Avoid anything containing alcohol (i.e., vinegar, mouthwash, aftershave, cough medication) while using disulfiram and for 2 weeks after discontinuing. c. A disulfiram-alcohol reaction can occur up to 1 week after discontinuing disulfiram. d. Disulfiram can lower liver function tests so monitoring is necessary. Which mood stabilizer is most associated with a potentially life-threatening rash? a. Divalproex (Depakote) b. Lamotrigine (Lamictal) c. Lithium (Eskalith) d. Carbamazepine (Tegretol) Which of the following statements most accurately reflects the predominant risk factors of antisocial personality disorders? a. Gang affiliation and early substance abuse b. Genetic predisposition of first-degree family member c. Childhood abuse and trauma from domestic violence d. Lower socioeconomic status from single-parent families A 37-year-old patient with schizophrenia is started on a high dose of chlorpromazine (Thorazine). The next day, he complains of lightheadedness after standing up. Vital sign changes reflect a significant decrease in blood pressure and an increase in heart rate. You suspect this is secondary to the alpha 1-adrenergic blockade which is causing a. Orthostatic hypotension b. Dry mouth c. Respiratory depression d. Decreased sweating A 74-year-old female presents for an evaluation. The psychiatric nurse practitioner has the patient complete the mini mental status examination and clock drawing. The patient is unable to correctly draw the face of the clock. Which area of the brain is likely to have a compromised functioning? a. Wernicke's Area b. Corpus Callosum c. Left Hemisphere d. Right hemisphere Your patient comes into the office for her middle of the year appointment. She is diagnosed with Bipolar I disorder. She has been maintained well for the past 3 years on Oxcarbazepine (Trileptal) 900 mg po QHS. Her labs are normal except for a blood sugar of 114 and her hemoglobin A1C is 5.6. She has a normal energy level and states that her mood is stable but complains of excessive urination and thirst. You suspect: a. Possible Diabetes 2 b. Possible hyponatremia c. Possible polydipsia d. Side effects of oxcarbazepine What is a rare but serious side effect of trazodone (Desyrel)? a. Confusion b. Priapism c. Rash d. Tremor When studying pharmacodynamics involving receptors, you know that an inverse agonist produces the following effect: a. Does not fully activate the receptor and causes only limited actions b. Causes the opposite effect of agonist, and causes the receptor to close the ion channel c. Blocks the agonist from opening the channel, and does not activate a biological response d. Activates a biological response, and opens the ion channel. Mr. Smith is a 56-year-old white male who has been successfully treated on Selegiline for over 4 years. Mr. Smith is going in for an elective surgical procedure. Which medication is strictly contraindicated with Selegiline? a. Non-steroidal anti-inflammatory drugs (NSAIDs) b. Meperidine c. Codeine d. Morphine Patty is a 62-year-old white female with Bipolar I Disorder and has been stable for 2 years on valproate and risperidone. She was recently diagnosed with shingles, and her primary care nurse practitioner started her on corticosteroids. As the PMHNP treating Patty, you are concerned that the addition of the corticosteroids may cause a. Neuroleptic Malignant Syndrome b. A hypertensive crisis c. A manic episode d. Stevens-Johnson syndrome Which serotonin receptor antagonism makes an antipsychotic "atypical"? a. 5HT4A b. 5HT1A c. 5HT3A d. 5HT2A What direct-acting dopamine receptor agonist is recommended to be used in the treatment of neuroleptic malignant syndrome (NMS) to help lower the dopamine receptor blockade? a. Benztropine (Cogentin) b. Bromocriptine (Parlodel) c. Trihexyphenidyl (Artane) d. Dantrolene (Dantrium) A 17-year-old female presents to your clinic reporting anhedonia, decreased energy, and hopelessness for the past two years. She denies drinking alcohol, smoking, or using illegal substances. Her physical examination results are within normal limits and she denies suicidal ideations. As the treating psychiatric mental health nurse practitioner, you decide to prescribe her bupropion (Wellbutrin). Of the following, which is contraindication for the use of bupropion? a. Bulimia b. Dysthymia c. Smoking d. Fatigue Which of the following statements reflects current understanding of neurodevelopment and best explains risk-taking behaviors of adolescent males? a. Hormonal flooding, especially testosterone and oxytocin, increase dopamine levels in the brain b. Maturation of the prefrontal cortex continues into mid-20s and early 30s c. Pruning of serotonin circuits in the adolescent brain is associated with increased impulsivity d. Stimulant and substance use impairs judgment and cognitive appraisal. Which of the following statements does not reflect current understanding of neurotransmitter pathways implicated in anxiety disorders? a. Decreasing norepinephrine in the locus coeruleus diminishes symptoms of anxiety. b. Increased levels of CRF in the amygdala, hippocampus and locus coeruleus increases symptoms of anxiety. c. Decreasing GABA in the mesolimbic cortex diminishes symptoms of anxiety. d. Increasing serotonergic activity in the amygdala diminishes symptoms of anxiety. Which of the following findings on MRI and PET scans would be uncommon in individuals with schizophrenia? a. Decreased glutamate and GABA release b. Decreased total brain volume c. Decreased electrical activity in the frontal lobes d. Decreased size of cerebral ventricles Which of the following findings would raise concern in a pre-school wellness exam for a 5-year-old girl? a. Blood pressure 92/52 b. Pulse 102, respirations 30 c. Completed immunizations: MMR, Varicella, DTap, IPV, PCV, Hib, HepB d. Height 43 inches, weight 55 lbs The PMHNP would expect to see which of the following during a physical exam for a patient with bulimia nervosa? a. Obesity b. Tachycardia c. Alopecia areata d. Erosion of dental enamel 1. While treating a 12-year-old boy for ADHD, the PMHNP observes which of the following physical features that raise concerns for genetic evaluation for Fragile X syndrome? a. Small head, short palpebral fissure, inner epicanthal folds b. Café au lait spots on face and arms, neurofibromas c. Long head and ears, short stature, hyperextensible joints d. Obesity, small stature, small hands and feet, hypotonia You are on call for your service. You arrive at the emergency department to find a 22-year-old male patient complaining of having trouble with his eyes. His vitals are stable. He is 5 foot and 7 inches tall; his weight is 140 pounds; his labs are all in the normal range. He states that he has been taking Aripiprazole (Abilify) for the last 2 months for his Bipolar 1 disorder. He had the dose raised to 15 mg po QAM 4 days ago to manage a manic episode. He states that one to three times a day for the past 2 days his eyes will roll upwards for 10 to 15 minutes and he will have trouble seeing until they roll back down. When you examine his eyes, he is able to fully open his eyes but he is not able to focus his eyes on you. His eyes are rolled upwards about 2/3rds of the way towards his upper eyelid. After examination you decide to: a. Discontinue the Aripiprazole and begin the patient on a gradually increasing regimen of Lamotrigine 25 mg 1 po QHS for 1 week. Have him return to his provider for further management of his bipolar disorder. b. Continue the Aripiprazole and give Benztropine Mesylate (Cogentin) 1 mg IM. Have him take Benztropine Mesylate (Cogentin) 1 mg po BID. Have him return to his provider for further management of his bipolar disorder the next day. c. Begin a cross titration downward of the Aripiprazole and upwards of Haloperidol (Haldol). Have him return to his provider for further management of his bipolar disorder. d. Discontinue the Aripiprazole and give Benztropine Mesylate (Cogentin) 1 mg IM. Have him take Benztropine Mesylate 1 mg po BID for 5 days. Have him return to his provider for a different medication for his bipolar disorder the next day. A 24-year-old mother of a 2-year-old child presents to the clinic. Which of the following facial characteristics would lead the psychiatric mental health nurse practitioner to believe the child has fetal alcohol syndrome? a. Large head, thick upper lip, and overdeveloped jaw b. Small head, thin upper lip, and smooth philtrum c. Long nose, large eyelid opening, and developed philtrum d. High nasal bridge, plump midface, and large eyelid opening A client is suffering from schizophrenia. Which of the following would be the appropriate question for the PMHNP to ask when assessing side effects produced by dopamine antagonism in the tuberoinfundibular pathway? a. Are you experiencing breast discharge? b. Are you experiencing drooling? c. Are you experiencing tremors? d. Are you experiencing dizziness? Which of the following patients with generalized anxiety disorder is most likely to be a CYP2C19 poor metabolizer and have an exaggerated response to diazepam (Valium) 5 mg TID with increased sedation, central nervous system, and cognitive side effects? a. 40-year-old Asian male b. 20-year-old African American female c. 50-year-old Caucasian male d. 30-year-old African American male Norepinephrine is a neurotransmitter that is implicated in alertness and anxiety. What areas of the brain has a large majority of norepinephrine neurons? a. Locus Coeruleus b. Amygdala c. Nucleus Accumbens d. Hippocampus When assessing an elderly patient who complains of tinnitus, the PMHNP would associate which of the following findings with an acoustic neuroma? a. Inflammation of middle ear b. Perforation of tympanic membrane c. Suppuration of the labyrinth d. Unilateral ringing in ear Which of the following medications has a unique mechanism of action in that it is both a dopamine and norepinephrine reuptake inhibitor? a. Bupropion (Wellbutrin) b. Imipramine (Tofranil) c. Venlafaxine (Effexor) d. Duloxetine (Cymbalta) Which of the following statements reflect the current understanding of dopamine (DA) pathways and clinical symptoms in schizophrenia? a. Negative symptoms are related to DA deficit in the mesolimbic system; positive symptoms are related to DA excess in the substantia nigra and ventral tegmental area b. Negative symptoms are related to DA excess in the cerebral cortex; positive symptoms are related to DA deficit in the nucleus accumbens and mesolimbic system. c. Negative symptoms are relat4ed to DA excess in the mesolimbic system; positive symptoms are related to DA deficit in the substantia nigra and ventral tegmental area. d. Negative symptoms are related to DA deficit in the cerebral cortex; positive symptoms are related to DA excess in the nucleus accumbens and mesolimbic system Arcus senilis when seen before the age of 40 years, is associated with which of the following disorders? a. Hyperlipidemia b. Hypertension c. HIV infection d. Syphilis Which cytochrome (CYP) enzyme is implicated as a tobacco inducer when an individual is treated on clozapine? a. 1A2 b. 2C9 c. 2D6 d. 2C19 A 43-year-old male is treated for Bipolar I on lithium. Which of the following hematologic changes is associated with lithium? a. Leukopenia b. Anemia c. Agranulocytosis d. Leukocytosis If given during pregnancy, lithium can cause which of the following medical problems in the baby? a. Spina bifida b. Ebstein's anomaly c. Neural tube defect d. Cleft palate Sandy, a 32-year-old separated female presents for an initial intake. Sandy states she recently left an abusive relationship, is living alone, and is experiencing fear, panic, and extreme anxiety. Which brain structure is activated in patients with severe anxiety, as expressed by Sandy? a. Thalamus b. Hippocampus c. Amygdala d. Cingulate gyrus The risk of bleeding on SSRI, non-steroidal anti-inflammatory drugs (NSAIDs), or aspirin, is an example of which of the following principles? a. Pharmacodynamic b. Absorption c. Distribution d. Pharmacokinetics You have a 17-year-old female patient who suffers with bipolar disorder. The patient has a history of impulsive acting out and promiscuity. You make a decision to add folic acid 0.8mg to her medication regimen of lamotrigine (Lamictal) 200 mg po QHS and Quetiapine XR (Seroquel XR) 50 mg 1 po QHS. Why is this important? a. The folic acid is important for the continuing development of the patient's neuroendocrine system. b. The folic acid is important to help her with the sleepiness induced by her bipolar medications. c. The folic acid is important to help her with her hypotension induced by the Quetiapine XR (Seroquel XR) 50 mg po QHS. d. The folic acid supports neural tube development during the first month that a woman is pregnant. ANCC Domain 2: Advanced Practice Skills (49 questions & Answers) 2022. A patient comes into your office inn a full manic episode. This patient has a diagnosis of Bipolar and Alcohol Use Disorder, moderate. You have a report from the patient's chemical dependency counselor indicating that inpatient chemical dependency counseling is needed. Your patient has been on the fence about inpatient chemical dependency treatment in the past. You assess the patient and find a strong odor of alcohol, auditory and visual hallucinations, flight of ideas, lack of sleep for 3 days, suicidal ideations without a firm plan, and plans to drive to another state to gamble. The patient's brother who accompanies him corroborates this information. In order to best treat this patient you: a. Admit him to the dual diagnosis chemical dependency unit with an order for the collection of laboratory values. b. Admit him to the inpatient chemical dependency program. c. Send the patient home with his brother and a prescription for a new antipsychotic medication. d. Admit him to the local psychiatric unit, with an order for the collection of laboratory values. Appraisal of the patient's suicidal ideation, plan, method, intent, and access to implement plan would be documented in which part of the standard psychiatric evaluation? a. History of Presenting Illness b. Review of Systems c. Diagnosis d. Mental Status Exam Violence directed toward women by an intimate partner is a serious health problem worldwide. Which of the following women has the greatest risk factors for being the victim of violence? a. 26 years old, recently separated, Asian American woman living in Boston, Massachusetts b. 42 years old, divorced, Caucasian woman dating a recently divorced man in Denver, Colorado c. 17 years old, single, Native American woman living with employed boyfriend in Galena, Alaska d. 35 years old, pregnant, African American woman living with her husband in suburb of Dallas, Texas Which of the following statements would be most likely to be effective in changing sexual behavior when counseling a sexually active 15-year-old adolescent girl? a. Always use condoms when having sex. b. Use this sample condom c. Require your partner to use a condom d. Always carry condoms in your purse. A PMHNP is working in an integrated care clinic and his family nurse practitioner colleague asks for a consult. The family nurse practitioner asks the PMHNP what antidepressant to start for a patient who has a score of 15 on the PHQ-9. What is the appropriate response? a. A score of 15 on the PHQ9 indicates moderate depression, so the patient should be started on a selective serotonin norepinephrine reuptake inhibitor. b. A score of 15 on the PHQ9 indicates severe depression, so you should start a SSRI and an atypical antipsychotic c. A score of 15 on the PHQ9 indicates moderate depression, so y9ou can refer the patient to therapy and start an antidepressant. d. A score of 15 on the PHQ9 indicates possible moderate depression, but a follow-up interview is required to assess for depression. I can see this patient today and further assess for depression and the appropriate treatment. A 10-year-old child lost her father to an unexpected heart attack. Normally confident, sociable and an excellent student, her teachers and mother report that she has withdrawn, become disinterested in schoolmates and schoolwork, and easily becomes tearful if anyone mentions her father. What non-pharmacological intervention would be most helpful at this time to facilitate her grieving and loss? a. Bibliotherapy b. Play therapy c. Individual therapy d. Psychoeducation group A mother presents to the clinic with her 9-year-old daughter because she is worried about her. The mother tells you that her daughter has been waking up in the middle of the night screaming off and on for the past several months. When she goes into her daughter's room, she is sitting up in bed and is initially inconsolable but eventually will go back to sleep. The mother is crying and tells you she is unable to go back to sleep after her daughter's episodes and feels exhausted all of the time. Which is the most appropriate initial response? a. I know you are upset, but it will get better b. Are you worried that this is your fault? c. I can see this is very upsetting for you d. Tell me why you are unable to fall back asleep after your daughter does In assessing possible sexual abuse of a 4-year-old child, which of the following would not be indicated? a. Has anyone ever taken your picture without your clothes on? b. Has anyone else asked you to take off your clothes? c. Has anyone invited you to a birthday pool party? d. Has anyone done something you didn't like to your body? A 78-year-old cattleman referred for treatment of refractory depression by his primary care provider reports continued thoughts of failure, lack of purpose in life, lack of interest in ranching, fishing, hunting, or family since losing his driver's license due to DUI six months ago. Which of the following areas is an assessment priority? a. Extent of alcohol use and motivation to reduce to safe levels b. Prior and current medications, dose, clinical response, side effects c. Sleep patterns (early-middle-late awakening), sleep hygiene d. Thoughts of self-harm, plan, intent, access During an initial psychiatric evaluation, the patient reports use of alcohol and illicit drugs. This information would be most appropriate in which part of the evaluation? a. Review of Systems b. Personal and social history c. Chief complaint d. Mental Status Exam The American College of Obstetricians and Gynecologists (ACOG) recommends screening for postpartum depression with a validated screening tool. Which of the following is a validated screening tool which has been recommended by ACOG? a. Major Depression Inventory (MDI) b. Montgomery-Asberg Depression Rating Scale (MADRS) c. Edinburgh Postnatal Depression Scale (EPDS) d. Hamilton Rating Scale for Depression (HRSD) A 67-year-old Vietnamese immigrant, who has been a resident in the U.S. for 3 years, comes to your clinic for an evaluation. Her first language is Vietnamese, but she does speak some English. She has problems with reading and writing, as she did not finish high school, or earn a GED (General Educational Development) equivalency. As the PMHNP, what is your priority disparity to address with this patient? a. Age b. Health literacy c. Reading level d. Race An adult female patient has been prescribed an SSRI for an initial episode of major depression. She is fearful of side effects and of becoming suicidal. Which of the following initial responses is most appropriate? a. SSRI's are generally quite safe and rarely cause suicidal thoughts. I could call you every few days to see if you are having any of these thoughts. b. SSRI's are the safest antidepressant medications and rarely cause suicidal thoughts. Try the medication and we can watch closely for any side effects. c. Have you ever had feelings of hurting yourself? If you took the medication and had thoughts like that, would you be willing to call the crisis hotline? d. Have you ever had feelings of hurting yourself? If you took the medication and had thoughts like that, what would you do? The Mini-Mental Status Exam (MMSE) is a brief, standardized screening tool designed for use: a. Across the lifespan to document mental status b. In older adults ³ 65 years to quantify cognitive status c. In adults ³18 years to document mental status d. In adults to quantify cognitive status. A patient who works the night shift presents for a follow-up appointment to see the PMHNP and states, "I can't sleep when I get home from work." Upon assessment, the PMHNP learns the patient recently started an exercise regimen and is now exercising after work. Which response about sleep hygiene should the PMHNP provide to the patient? a. Exercising is a good way to help you get sleep, so keep up your schedule and your body will adjust to your new routine. b. Regular exercise can help promote sleep but should not be performed too close to your bedtime or it can interfere with sleep; try exercising before work. c. Exercise dose not promote or help deepen your sleep, so you should consider making changes in your daily routine. d. Regular exercise helps with sleep, so continue to exercise, and when you get home, watch television to help promote sleep. A 36-year-old woman was fired from her job as a paralegal at a law firm. She had worked for the same attorney for 10 years. As soon as she came in the door from work, her husband noticed she appeared upset and asked her if she was alright. She immediately started yelling at her husband because he had not given their 5-year-od daughter a bath and had not cleaned the house. Which of the following defense mechanisms is being demonstrated by this woman? a. Dissociation b. Conversion c. Rationalization d. Displacement The Five A's for Health Behavior Change include the following: a. Assess, Appraise, Advise, Act, Assist b. Assess, Appraise, Agree, Access, Arrange c. Assess, Advise, Act, Access, Assist d. Assess, Advise, Agree, Assist, Arrange Dementia screening for the general population is not recommended for the following reason: a. The service benefits few people: target condition has high prevalence in the chosen population b. The service is unfocused for target condition: sensitivity and specificity of dementia screening measures do not currently warrant widespread use. c. The service has unknown balance of benefit and harm: current treatments have limited effectiveness in modifying instrumental activities of daily living. d. The screening causes net harm in the target population: time involved in screening detracts from higher priority preventive health measures in general population. What opening question or statement by a PMHNP facilitates communication at an initial visit? a. Tell me what you think the problem is b. What seems to be the problem today? c. What problem does your primary care physician want us to evaluate? d. Tell me your concerns and how I may help. When assessing a client, what is the most important predictor of potential violence? a. History of substance abuse b. History of seizures c. History of aggressive rage d. History of head injury As the PMHNP working with a young female recently diagnosed with bipolar disorder and using a recovery model, what will be the focus of your interactions with this patient? a. Psychoeducation about her disease process b. Assisting in gainful employment c. Focus on the side effects of her medications d. Focus on her feelings, experience, and what she wants to achieve A 32-year-old Caucasian Catholic female patient presents for her therapy session. She is upset because she and her husband have been in an argument over finances. She does not work and stays at home to take care of the children. She reports feeling sad and has had thoughts of hurting herself. She has a past history of overdosing with Tylenol several years ago. Which of the following places her at high-risk for suicide? a. Age b. Previous suicide attempt c. Marital status d. Gender During an initial assessment, the PMHNP asks the patient to perform the serial 7's. What aspect of the mental status exam would the nurse practitioner be assessing? a. Memory b. Concentration c. Speech d. Affect Which of the following illustrates the role of the PMHNP in reducing the stigma of mental illness through community education as primary prevention? a. Professional speaker interviewed by morning television talk show anchor on depression. b. Expert witness for standard of care regarding psychiatric nursing staff in wrongful death lawsuit c. Teaching high school class about depression and teen suicide d. Volunteering to help with depression screenings at a community health fair - A In a psychiatric emergency, the PMHNP is aware that the most important goal of the interview is self-protection. What can the PMHNP do to keep himself or herself safe? a. Make sure the patient is restrained before beginning the evaluation b. Enter the room alone, in order to build a therapeutic alliance c. Know as much about the person as possible prior to starting the interview d. Inform the patient that violence is not acceptable. When questioning a patient regarding alcohol use during an intake, the patient t3ells you he only drinks socially. Which initial response is most appropriate? a. What amount and what kind of alcohol do you drink in a week? b. I am glad you are a responsible drinker. c. Many people with alcohol issues say they are social drinkers. d. Do other people in your household drink alcohol? - A During a follow-up medication visit with a 13-year-old male, his mother stayed longer to ask the PMHNP a question. Specifically, his mother states, "The school sent home information about the human papillomavirus and suggested my son see his primary care provider to get an immunization. Why did I get this information for my son? Furthermore, he is not sexually active." Which of the following would be the PMHNP's best response? a. I understand your concern with talking to your son about becoming sexually active. Let's address how to start the conversation. b. You are correct, only females need to receive the human papillomavirus vaccination. This letter must have been sent to you by mistake. c. Decisions about immunizations can be difficult. The Centers for Disease Control and Prevention recommend that all males between the ages of 11-12, or as early as 9 years of age, receive this vaccination. What concerns do you have? d. As your son's PMHNP, I cannot provide you with information about immunizations. The World Health Organization has predicted that depression will be the number one world health problem in the 21st century. Lifetime prevalence for major depressive episode is 12% (with a 5-17% range). Which of the following individuals has the greatest risk factors for MDD? a. 16-year-old, high school male, honors student, who runs cross-country track b. 28-year-old, single woman, working toward partner in a large law firm c. 43-year-old, divorced woman, schoolteacher, raising three children d. 58-year-old, married male, computer analyst, whose work requires frequent travel Which of the following interventions has greatest relevance when counseling parents of a 20-year-old male patient who has recently been diagnosed with schizophrenia? a. Family therapy to facilitate gradual independence for the patient while addressing parental desire to protect their son b. Patient and family education about the disease course, treatment regimens, support systems, and life management skills c. Multi-system family therapy that includes in-home therapy as well as multiple family group therapy to facilitate shared learning from other families with members with schizophrenia d. Supportive psychotherapy group for the parents to facilitate their grief and build coping skills to help manage their son's life-changing mental illness. A patient was started on an antipsychotic medication two weeks ago and is now experiencing the following symptoms: inability to sit still, pacing, and feelings of inner restlessness. He does not complain of any anxiety or being worried about anything in particular. Which of the following rating scales would be the most appropriate to assess these symptoms? a. Abnormal Involuntary Movement Scale (AIMS) b. Barnes Akathisia Rating Scale (BARS) c. Dyskinesia Identification System d. Simpson Angus EPS Scale Which of the following screening tests has high sensitivity and specificity for identifying cognitive impairment in older adults while requiring the least amount of time to administer in a clinical setting? a. Global Deterioration Scale b. Clock Drawing Test c. Mini-Mental Status Exam d. Dementia Rating Scale The PMHNP understands which factor to be the most important in therapeutic communication: a. Verbal communication b. Feedback c. Clarification d. Nonverbal communication In evaluating the functional status of a 68-year-old woman who has depression, the Instrumental Activities of Daily Living Scale includes which of the following activities? a. Toileting b. Filing taxes c. Using telephone d. Bathing Which of the following lifestyle factors poses the greatest risk for depression, irritability, liver problems, and hypertension in a college student? a. Binge drinking b. Marijuana use c. Anabolic steroid use d. Cocaine use A female client has gradually assumed increasing responsibility for the care of her aging parents. Her mother was diagnosed with Alzheimer's type dementia three years ago and is increasingly disoriented to her home surroundings. Her father insists on keeping her mother at home to provide care until she dies. What is the first priority in counseling this couple and their caretaking adult daughter? a. Anticipatory guidance b. Caregiver strain c. Patient safety d. Patient and family education There is a growing recognition that adverse life experiences due to a variety of reasons underlie a wide range of psychiatric disorders and medical problems. According to a seminal research study on adverse childhood experiences (ACE), a positive relationship exists between ACE and which of the following disorders? a. Alcoholism, anxiety, heart disease, sleep disorders, and schizophrenia b. Alcoholism, anorexia, depression, diabetes, and schizophrenia c. Alcoholism, anxiety, depression, diabetes, and schizophrenia d. Alcoholism, anxiety, depression, diabetes, and heart disease A 44-year-old man presents to the PMHNNP at his wife's insistence. The patient was hospitalized for a mild head injury after a motor vehicle accident four weeks ago. He reports a 3-year history of drinking alcohol. The patient states he was at the store a week ago when he realized the store clerk was an imposter replacing his wife. The patient's wife insists that her husband is delusional and she intends to file for divorce if he does not get help. Which of the following types of delusions is the patient expressing? a. Fregoli phenomenon b. Capgras syndrome c. Delusion of doubles d. Clerambault syndrome In advising parents how to reduce the risk for teen substance abuse when there is positive family history of alcohol dependence, a PMHNP encourages all of the following recommendations except: a. Acknowledgement of family risk factor and open discussion with teen b. Parental presence and involvement in child's school and sports activities c. Random urine testing using home drug testing kits d. Parental networking and supervision of any teen parties. In assessing patient risks for binge drinking, which of the following statements is not accurate? a. Binge alcohol use rates are similar across different levels of education b. Binge drinking is higher in urban, densely populated areas than rural, sparsely populated areas. c. Whites have a higher rate of binge drinking than black or Hispanics. d. Men are much more likely than women to be binge drinkers. A 14-year-old girl admitted to a psychiatric unit following an overdose on alcohol and benzodiazepines after the breakup of a relationship has made seductive comments to her roommate, which is upsetting her roommate. In assessing the girl's behavior toward her roommate, the PMHNP needs to include the topic of sexual orientation. What is the best introductory approach to this topic? a. Are you sexually active? b. Are you attracted to girls rather than boys? c. What is your experience in sexual relationships? d. Did you overdose after breaking up with a girlfriend or boyfriend? An example of a screening tool that measures severity and tracks change in specific symptoms is a. The CAGE b. The Mini-Mental State Examination (MMSE) c. The Short Form 36 (SF-36) d. The Behavior and Symptom Identification Scale (BASIS) Which of the following risk factors is not amenable to lifestyle changes in reducing lifetime risk of depressive disorders? a. Alcohol use b. Stressful family life c. Personality d. Work-related stress A 31-year-old married woman with a history of anorexia nervosa, binging-purging type, during her teens and early 20's, is pregnant. She has had two prior spontaneous abortions in late 20's. She is worried whether this pregnancy may trigger recurrence of her prior eating disorder, adversely impact fetal development, or affect the likelihood of full-term delivery. At 5'7" she maintains a weight of 120 pounds by running 6-10 miles daily as conditioning for marathons and a 2,000 calorie daily vegan diet. What lifestyle modifications would reduce risks of exacerbating an eating disorder and increasing likelihood of normal fetal development full-term delivery. a. Increase daily caloric intake by 1,000 calories, continue running as tolerated with reduction in second or third trimester, add pre-natal vitamin, vitamin E supplement, and B-complex supplement, plan 20-30-pound gradual weight gain over the course of pregnancy. b. Increase daily caloric intake by 500 calories, reduce running to 2-4 miles daily, alternate with stretching and yoga in second and third trimesters, add prenatal vitamin and vitamin B-12 supplement, plan a 28-40-pound gradual weight gain over the course of pregnancy. c. Increase daily caloric intake by 250 calories, reduce running to 4-8 miles daily with reduction in second or third trimester, add prenatal vitamin, Calcium supplement, and plan a 20-30-pound gradual weight gain over the course of pregnancy. d. Increase daily caloric intake by 700 calories, continue running as tolerated with reduction in second or third trimester, add prenatal vitamin E supplement, and B-complex supplement, plan a 25-35-pound gradual weight gain over the course of the pregnancy. In the aftermath of a tornado that destroyed over 100 homes in a rural Midwest community, the PMHNP draws on principles of crisis intervention while working with an elderly couple who lost everything in the storm during the night. Which of the following statements is most appropriate at the initial encounter? a. "What medications were you taking? Let me work on that first." b. "Tell me everything that happened. We'll sort through the next steps." c. "What is your biggest concern right now? I will help you as much as possible." d. "You will be able to stay in the shelter until the FEMA trailers arrive. It's going to be OK." When child, adult partner, or elder abuse is suspected, the PMHNP needs to conduct an abuse assessment screen in privacy, away from the partner, the child's parents or guardians, or the elder person's relative or companion. Simple, direct questions in a nonjudgmental interview are indicated. Which of the following would be an appropriate abuse screening question? a. Within the past year, have you been hit, slapped, kicked or otherwise physically hurt by someone? b. Within the past year, has anyone ever forced you to do something by threatening you with a gun? c. Within the past year, have you ever threatened or tried to commit suicide to escape the abuse? d. Within the past year, has anyone ever threatened to kill you? - A A PMHNP realizes she is frustrated and easily angered with one of her patients for no apparent reason. When discussing the situation with a colleague, she states the patient reminds her of her abusive stepfather. Which of the following best describes the nurse practitioner's reaction? a. Projection b. Displacement c. Transference d. Countertransference When assessing for alcohol use, what does the CAGE questionnaire ask? a. Have you ever felt that you should cut down on your drinking, do you get angry when you drink, do you feel good about your alcohol use, have you ever had an eye-opener in the morning to get rid of a hang over? b. Have you ever felt you should cut down on your drinking, have people annoyed you by criticizing your drinking, have you felt guilty about your drinking, have you ever had an eye opener in the morning to get rid of a hangover? c. Have you ever felt you should cut down on your drinking, do people get angry when you drink, do you feel good about your alcohol use, have you ever had an eye-opener in the morning? d. Have you ever felt you should cut down on your drinking, do you become avoidant when you drink, do you feel guilty about your alcohol use, have you ever had an eye opener in the morning to get rid of a hangover? Which of the following would not be advised to divorced parents to facilitate recovery for their children? a. The divorced couple must provide an age-appropriate truthful explanation why they divorced b. The divorced couple must show consistent behavior toward the child c. The divorced couple must avoid arguing with one another d. The divorced couple must continue to relate to the child despite the child's anger - A A 53-year-old male with a history of schizophrenia s being discharged from an inpatient admission to a psychiatric hospital and was referred for to you for follow up in your private practice. During your initial assessment with the patient, you ask him about his education level and he talks about where he went to high school, his high school friends, activities he involved in, and his high school graduation. He then states, "that was the end of my education." Which of the following does the patient's answer demonstrate? a. Loosening of associations b. Tangentiality c. Circumstantiality d. Perseveration ANCC IQ Domain 3: Diagnosis and Treatment (52 questions & Answers 2022) In distinguishing HIV-related depression from HIV dementia, which of the following findings would be indicative of HIV dementia? a. Apathy and inertia b. Incoordination c. Decreased motivation d. Diminished attention and concentration Clozapine is associated with severe neutropenia (absolute neutrophil count (ANC) less than 500/?L). The requirements to prescribe, dispense, and receive Clozapine are incorporated into a single, shared program called the Clozapine Risk Evaluation and Mitigation Strategy (REMS). Who implement the Clozapine Risk Evaluation and Mitigation Strategy (REMS)? a. A Federal Drug Administration (FDA) b. Centers for Disease Control and Prevention (CDC) c. Health and Human Services d. The manufactures of Clozapine (Clozaril) You have a 16-year-old patient who suffers from Binge Eating Disorder. She is 5 foot 10 inches tall (70 inches) and has a BMI of 30. You consult the National Guideline Clearing House for the best information on treating the patient. Part of your treatment plan includes: a. Implementing non-diet health at every size therapy in addition to psychotherapy to help with weight stabilization and binge eating b. Treatment with Duloxetine (Cymbalta) long-term at low doses and a reduced calorie diet. It is associated with significant weight loss. c. Urine specific gravity, orthostatic vital signs, and oral temperatures taken on a regular basis. d. Use of Sibutramine (Meridia) 10 mg po QAM and a reduced calorie diet. It is associated with significant weight loss. When evaluating a 5-year-old child with language deficits, which of the following is a key indicator in differentiating autistic disorder from a mixed receptive-expressive language disorder. a. Imaginative play is predominant form of expression b. Language abnormalities such as echolalia are common c. Family history of speech delay or language problems d. Level of intelligence ranges from mild to severe impairment A client says that because he wished his sister was dead, and your client's sister subsequently was killed in a motor vehicle accident, the death was caused by the client's wishes. The client also revealed that he can read other's minds. This client is most likely suffering from which personality disorder? a. Schizotypal Personality Disorder b. Schizoid Personality Disorder c. Paranoid Personality Disorder d. Delusional Personality Disorder Which of the following atypical antipsychotic medications has U.S. FDA approval for treatment of bipolar depression when combined with fluoxetine (Prozac)? a. Quetiapine (Seroquel) b. Olanzapine (Zyprexa) c. Aripiprazole (Abilify) d. Risperidone (Risperdal) The PMHNP treating a patient for schizophrenia on ziprasidone orders and EKG. Which QTc interval result places the patient at greatest risk for torsade's de points? a. 160-260 milliseconds b. 300-500 milliseconds c. 500-700 milliseconds d. 100-200 milliseconds A pervasive pattern of grandiosity, lack of empathy, and need for admiration suggests the diagnosis of which of the following personality disorders? a. Paranoid b. Borderline c. Schizotypal d. Narcissistic When counseling a 52-year-old perimenopausal woman with nocturnal hot flashes, anxiety, depressed mood, low energy, little motivation, diminished sex drive, and work-related stress, what lifestyle changes and complementary therapies would be evidence-based recommendations to consider when this patient does not want to be on hormone replacement or antidepressant medication? a. Yoga, isoflavinoids, and black cohosh b. Walking, St. John's Wort, and black cohosh c. Tai chi, dietary soy, and flaxseed d. Current evidence does not support the efficacy of complementary therapies for perimenopausal or depressive symptoms In managing the maintenance phase for Bipolar I Disorder, which of the following statements is not supported by current evidence in the literature? a. Lamotrigine, but not lithium, is superior to placebo in preventing a depressive episode. b. Extended-release formulation of carbamazepine is equivalent to lithium in preventing a manic, hypomanic, or mixed episode. c. Both lamotrigine and lithium are superior to placebo in delaying onset of mood-related episodes. d. Lithium, but not lamotrigine, is superior to placebo in preventing a manic, hypomanic, or mixed episode. Which of the following interventions does not have evidence-based findings that demonstrate reduced symptoms in individuals with non-combat related PTSD? a. Eye-movement, desensitization, and reprocessing (EMDR) b. Anticonvulsants c. Exposure-based cognitive behavior therapy (CBT) d. Selective Serotonin Reuptake Inhibitors (SSRIs) After two weeks of treating a client for psychosis, the client develops symptoms of neuroleptic malignant syndrome (NMS). The following factors enable the PMHNP to differentiate NMS from serotonin syndrome: a. Autonomic instability, diaphoresis, tremors b. Rigidity, hyperreflexia, orthostatic hypotension c. Mutism, leukocytosis, myoglobinuria d. Hyperthermia, leukopenia, tachycardia A 20-year-old male with no previous psychiatric history arrives at the emergency room with his college roommate. The roommate reports that the patient has been acting erratic for the past month, talking to people who do not exist, walking around naked, and accusing the roommate of spying on him. The patient's vital signs are WNL and his neur9ological examination does not show any abnormalities. The most important first laboratory test would be which of the following? a. Complete blood count b. Non contrast CT scan of the brain c. Liver function tests d. Toxicology test Mr. Ready is a 58-year-old new patient at a community mental health center. He's seen by the PMHNP for depression, anxiety, and co-morbid substance use disorder, which the patient describes as effective in treatment of his symptoms. He does not have a primary care provider and does not know when he last had laboratory values drawn. Ready takes Tylenol p.r.n. for his knee pain, Prilosec OTC 20 mg BID, and has been on his current psychotropic medications for over two years: Buspirone 20 mg BID, Sertraline 200 mg po daily, and Aripiprazole 2 mg daily. What screening labs are appropriate for this patient? a. BMP, LFT, and vitamin D b. Lipid Profile, CMP, CBC, Thyroid Profile, and Vitamins B12 and D c. CBC, CMP, Lipid Profile, and Vitamin B12 d. CBC and CMP In distinguishing borderline personality disorder from chronic post-traumatic stress disorder related to childhood sexual abuse and victimization, which of the following would be more indicative of borderline personality disorder? a. Feeling of detachment or estrangement from others b. Efforts to avoid real or imagined abandonment c. Affect lability often associate with intense fear d. Irritability or intense outbursts of anger Which of the following findings would raise concern in an annual exam of a 76-year-old woman? a. Fasting glucose 76 mg/dL; Hg 11.8 gm/dL; HCT 38% b. AST 85 u/L; ALT 45 u/L c. Height 65 inches; weight 140 lbs d. Blood pressure 130/86; pulse 82; respirations 25 An 8-year-old boy is referred to you by his school nurse because he has been complaining of stomach aches every morning in school for the past month. When interviewing the boy's mother, she states that he does not like to go to school, insists on coming home immediately after school, and sleeps with his parents at night. The mother denies any other complaints. Which of the following is the most likely diagnosis? a. Reactive attachment disorder b. Posttraumatic stress disorder c. Social phobia d. Separation anxiety disorder A PMHNP treating a 32-year-old woman after hospitalization for postpartum psychosis after the birth of her first child needs to consider which as the most important risk factor when monitoring her response to treatment? a. As many as two thirds of patients have a second episode of an underlying affective disorder during the year after the baby's birth b. Incidence of postpartum psychosis is about 1 to 2 per 1,000 childbirths and 50 t 60 percent of affected women have just had their first child c. Subsequent pregnancies are associated with increased risk of another episode, as high as 50 percent d. Data suggest that an episode of postpartum psychosis is essentially an episode of a mood disorder and is usually an indicator of bipolar disorder Which is the only medication that is approved by the U.S. FDA to treat depression in children? a. Paroxetine (Paxil) b. Sertraline (Zoloft) c. Fluoxetine (Prozac) d. Citalopram (Celexa) When suspecting a patient has NMS, which laboratory values would help confirm the diagnosis? a. Leukocytosis and elevated creatine phosphokinase b. Leukocytosis and thrombocytosis c. Leukopenia and decreased creatine phosphokinase d. Leukocytosis and thrombocytopenia A 34-year-old African American female presents for an initial psychiatric evaluation. On examination, she reports she thinks she has attention deficit disorder because she has a short attention span, poor recent memory, nervousness, and mood lability, and she is sweating more. Which laboratory test should the PMHNP order to rule out an organic etiology of her symptoms? a. CMP b. Thyroid profile c. Cortisol level d. Prolactin level Which lab test should be ordered to rule out a medical cause of dementia symptoms? a. Thiamine b. Vitamin B12 c. Albumin d. Vitamin D3 Making an evidence-based decision regarding implementation of an intervention includes current research evidence and all of the following except the: a. Patient's clinical status, circumstances, and preferences b. Generalizability of findings c. Clinician's expertise d. Availability of healthcare resources Which of the following is not characteristic of individuals with antisocial personality disorder (APD)? a. APD is five times more common in first-degree biologic relatives for men b. APD is strongly associated with alcohol and drug abuse c. APD is identified predominantly in European and Western industrialized countries d. APD is correlated with low dopamine levels in the frontal cortex implicated in aggression and impulsivity A WBC of 4,000 in a patient taking clozapine would prompt the PMHNP to take3 which of the following actions? a. Consult with hematologist to determine appropriate antibiotic regimen and monitor closely b. Discontinue clozapine, initiate alternative antipsychotic medication and monitor closely c. Institute twice-weekly complete blood count with differentials and monitor closely d. Institute daily complete blood count with differentials and monitor closely Your patient suffers from insomnia and bipolar disorder. She tells you that she likes to take her Lurasidone 20 mg (Latuda) at bedtime because it is easier for her to remember. She does not like to gain weight so she eats two 90-calorie packages of ham slices with the Lurasidone before she goes to bed. She complains of not being able to go to sleep very quickly. You respond: a. Improve your sleep hygiene, and continue to eat the ham at bedtime b. I will increase the Lurasidone dosage to 40 mg at bedtime to increase the soporific component c. I recommend that you take in 360 calories of carbohydrates at bedtime to induce sleepiness d. Increase the ham so that you are taking in 360 calories for full absorption An 88-year-old nursing home patient presents to the emergency department with recent mental status changes, including aggression, confusion, and dizziness, over the past two days. The patient scored a 20 on the Mini-mental status exam (MMSE). In addition to the MMSE, what additional information is needed to complete a thorough evaluation? a. Blood chemistry, complete blood count, and urinalysis b. Hamilton depression scale (HAMD) score c. Patient Health Questionnaire (PHQ-9) score d. Electrocardiography and pharmacogenomics testing A patient being treated for major depressive disorder and on sertraline (Zoloft), 150 mg po daily for the past 16 months, presents to the psychiatric mental health nurse practitioner for an outpatient follow-up visit. During the visit, the patient states she has not been feeling well, reporting the flu. She also states she has not taken her medication in the last five days. Which of the following symptoms would she be describing if you suspect SSRI discontinuation syndrome? a. Agitation, diaphoresis, tremor, and ataxia b. Restlessness, tremor, fever, and shivering c. Restlessness, headache, increased heart rate, and diarrhea d. Agitation, nausea, dysphoria, and disequilibrium For a patient who has a long history of opiate dependence and is newly in recovery, which of the following is most likely to aid in the treatment process? a. Acamprosate (Campral) b. Buprenorphine (Buprenex) c. Bupropion (Wellbutrin) d. Varenicline (Chantix) Lorazepam is the preferred benzodiazepine when managing alcohol withdrawal symptoms if a patient with co-morbid: a. OxyContin abuse b. Hepatic disease c. Seizure disorder d. Folate-deficiency anemia Following best practice guidelines, which pharmacologic and non-pharmacologic treatments are the best studied and have the highest level of evidence for the treatment of nightmares in adults with nightmares in adults with PTSD? a. Prazosin (Minipress) and image rehearsal therapy b. Quetiapine (Seroquel) and eye-movement desensitization and reprocessing (EMDR) c. Venlafaxine (Effexor) and hypnosis d. Clonidine (Catapres) and progressive muscle relaxation A patient presents for a clinic appointment and tells the PMHNP, "My depression is a little better, but I read about acupuncture, and I want to add this to my treatment." What is the PMHNP's best response to this patient? a. Integrative therapies have no effectiveness in improving depressive symptoms, ad if your depression has not improved ad you have not achieved remission of your symptoms, I recommend that we increase your antidepressant b. Acupuncture is one of many integrative therapies, but it has the least effectiveness in improving depression c. Integrative therapies such as acupuncture are not well studied, and I cannot recommend that you add this to your current treatment d. Acupuncture has been found to help some individuals with depression. While the data is limited, if this is something you would like to consider, let's talk about where you might find a reputable practitioner for acupuncture treatments In distinguishing between dementia and pseudodementia which of the following findings would be expected in a patient with pseudodementia? a. Struggle to perform tasks b. Nocturnal accentuation of dysfunction common c. "Don't know" answers typical d. Attention and concentration usually faulty Sally is a 27-year-old attorney who recently moved to your area. Sally presents with social anxiety disorder, specifically symptoms of performance anxiety. Sally's only other medical condition is exercise-induced asthma, and she is treated on Albuterol. Sally states she was in cognitive behavioral therapy (CBT), without relief, and would like to try a medication. Which is the most suitable initial treatment for Sally? a. Sertraline (Zoloft) b. Inderal (Propranolol) c. Risperidone (Risperdal) d. Clonazepam (Klonopin) A new patient comes to you on a medication regimen of Adderall XR 30 mg 1 po QAM, Seroquel 300 mg po QHS, Mirtazapine 45 mg po QHS, Diazepam 5 mg TID, Zolpidem 10 mg po QHS and Zolpidem 10 mg po QHS in addition if initial dose does not help the patient to sleep. You confirm the medication regimen with the patient's past records. You diagnose the patient with PTSD, Panic disorder, MDD, ADHD, and alcohol use disorder-moderate by history in recovery. You treat the patient and find that the patient is resistant to changing the past medication regimen. The patient calls you stating that the pharmacy will not refill the Zolpidem at this time. The patient wants you to rewrite the script so that the medication is available. In checking with the pharmacy, you find out that the patient has potentially used 60 tablets of Zolpidem in a 20-day period. Your best action is to: a. Refill the order for the medication. Then, send the patient for a chemical dependency evaluation. The patient is on too many sedative-type medications and is harming himself. b. Discontinue the prescription for the Zolpidem. Talk to the patient about the overuse of Zolpidem and the danger it poses to his health and wellbeing. c. Refill the order for the medication as requested. This medication has not harmed the patient so far. d. Give an order for the Zolpidem. Talk with the patient about the danger of using too much sleeping medication. Then, discontinue the medication. A 33-year-old female patient has been diagnosed with bipolar disorder. Before starting this patient on lithium (Eskalith) for mood stabilization, which of the following laboratory tests are appropriate to obtain for this patient? a. Thyroid function tests, creatinine, complete blood count b. Thyroid function tests, creatinine, pregnancy test c. Thyroid function tests, liver function tests, pregnancy test d. Thyroid function tests, creatinine, liver function tests Which of the following antipsychotic medications has dosage formulations which include oral tablets, orally disintegrating tablets, short-acting injections, and long-acting injections? a. Olanzapine (Zyprexa) b. Ziprasidone (Geodon) c. Paliperidone (Invega) d. Clozapine (Clozaril) A patient with a diagnosis of schizophrenia has a history of suicidal ideation and suicide attempts. The PMHNP should consider which antipsychotic medication that is the only known antipsychotic to reduce the risk of suicide in schizophrenia? a. Latuda (lurasidone) b. Abilify (aripiprazole) c. Clozaril (clozapine) d. Invega (iloperidone) Mr. Jones, a 78-year-old male patient, presents to the PMHNP for a follow-up medication appointment. Mr. Jones' depression has been successfully treated with citalopram 20 mg by mouth daily. During the visit, Mr. Jones complains that in the last 2 or 3 weeks, he has had nausea, fatigue, feeling weak, with a headache and decreased appetite. Which action would be the most appropriate for Mr. Jones? a. Assess for other symptoms of hyponatremia and check a serum sodium level b. Discuss rates of relapse on SSRI and recommend switching his medication to a different antidepressant class c. Discuss rates of relapse on an SSI and increase his medication to citalopram 40 mg by mouth daily d. Discuss SSRI discontinuation syndrome and stress the importance of medication adherence. When evaluating a woman with major depressive disorder, which of the following would not be a major consideration in the differential diagnosis a. Prior pregnancies b. Any episodes of hypomania c. Prior episodes of depression d. Any periods without sleep yet no fatigue Which of the following interventions have evidence-based findings to reduce co-morbid substance use in patients with schizophrenia? a. Voucher-based behavioral reinforcement b. Manualized group intervention with motivational enhancement c. Three-session individual motivational enhancement d. Psychosocial support group targeting coping skills In bipolar I disorder, men are more prone than women to have: a. Rapid cycling b. Manic episodes c. Mixed episodes d. Depressive episodes The PMHNP evaluates a 3-year-old male and makes the diagnosis of attention deficit disorder Following the FDA's labeling of approved medication for children, which medication can be started in a child at age 3? a. Fluoxetine b. Amphetamines c. Methylphenidate d. Bupropion Electroconvulsive therapy (ECT) is a treatment for depression. What adverse effect is uncommon for a patient to experience after receiving ECT treatments? a. Fractures b. Memory disturbances c. Headaches d. Muscle soreness In the American Journal of Psychiatry, you read an article that discusses a recent study's evidence that the nicotine exposure in pregnant women increases the risk for schizophrenia in offspring (i.e., 38% increased likelihood of schizophrenia in young adults whose mothers smoked heavily while pregnant). This inspires you to translate this information in practice and teaching roles as a nurse practitioner. What considerations are needed to translate this in to practice based on the best evidence? a. If the research is published in a peer-reviewed journal such as the American Journal of Psychiatry, it is sound research b. Analyze the original study only to determine that the research is sound and meets the criteria for effectiveness of an intervention c. Determine if any other psychiatric or medical journal articles have commented on the validity or generalizability of the original study d. Analyze the original and repeat studies for methodical rigor and verify that there were randomized and non-randomized clinical trials with the same results. You are seeing a female patient who is morbidly obese. She also suffers from major depression, anxiety, and antidepressant-induced decreased libido. She has been stabilized on desvenlafaxine (Pristiq) 50 mg 1 po QAM and bupropion (Wellbutrin) 50 mg. She also takes Mononessa (Ortho-cyclen) 1 po QAM and Vitamin D 1000 IU po QAM. She has had trouble maintaining weight loss on a diet support programs, diet medications, and self-directed food reduction plans. She asks you about using Phentermine to help her lose weight. You discuss options with her and decide to: a. Prescribe a one-month course of phentermine (Adipex-P) 37.5 mg po QAM. Have her come back in one month for a checkup. b. Discontinue her current bupropion. Prescribe bupropion and naloxone (Contrave) 90mg/8mg for one week. Have her come back in one week for a checkup. You may have to increase the dosage at one-week intervals. c. Prescribe a one-month course of Phentermine and Topiramate (Qsymia) 7.5/46 for 30 days. Have her return to the clinic in 1 month for a checkup. d. Prescribe a 1-month course of Topiramate (Topamax) 50 mg 1 po QHS. Have the patient come back in 1 month for a checkup. Let the patient know that you may have to increase the dosage at one-month intervals. In distinguishing between cortical and subcortical dementias, which of the following findings would be indicative of subcortical dementia? a. Late dysarthria b. Early aphasia c. Euthymic mood d. Early gait disturbances You are seeing a client for an initial psychiatric evaluation. The client complains of persistent irritability, racing thoughts, decreased need for sleep, distractibility, episodes of unusually high energy, and auditory hallucinations. The episodes of disturbed mood last approx.. 5 days and the psychotic symptoms occur even if mood is euthymic. The most likely diagnosis is: a. Bipolar I disorder with psychotic features b. Schizoaffective disorder bipolar type c. Bipolar II disorder with psychotic features d. Schizophrenia A middle-school boy repeatedly lies, steals, bully's peers, instigates fights, and inflicts harm without feeling any regret or remorse. These are characteristics of which disorder? a. Conduct disorder b. Oppositional defiant disorder c. Disruptive behavior disorder d. Antisocial personality disorder Your 37-year-old female patient suffers from schizophrenia. She has been maintained in stable condition for four years on Quetiapine (Seroquel). She tells you that her gynecologist has recommended that she have a total hysterectomy. You asked her if her gynecologist plans to prescribe hormone replacement after the total hysterectomy. She responds "no". You initiate a call to the gynecologist to discuss hormone replacement therapy a. The abrupt loss of estrogen from the ovaries can precipitate deterioration of mental state, hallucinations, delusions, and depression as well as the stress from surgical menopause. b. Consultation with the gynecologist is not needed. The hyperprolactinemia caused by the antipsychotic, Quetiapine, has already reduced estrogen production in the patient's body to a point where it is not an issue after four years on the medication. c. Consultation with the gynecologist is not needed. The patient can adapt to surgical menopause as easily as she would natural menopause. Estrogen waxes and wanes in the body naturally and plays no part in schizophrenia. d. You want to know if the gynecologist will wean the patient off her own estrogen. By suppressing the ovaries, she can be tapered off. Risk reduction for a 22-year-old patient with schizophrenia taking olanzapine includes patient education on daily exercise, balanced nutrition, and baseline and periodic monitoring of BP, weight, BMI, and: a. Fasting lipid panel, EKG, liver enzymes b. Fasting glucose, fasting lipid panel, electrolytes c. Waist circumference, fasting lipid panel, EKG, prolactin level d. Waist circumference, fasting glucose, fasting lipid profile. Which of the following lab findings would raise the greatest concern when prescribing lithium? a. BUN 20 mg/dL b. Serum creatinine 3.0 mg/dL c. Serum Na+ 120 mEq/L d. Glomerular Filtration Rate (GFR) 115 mL/min ANCC IQ Domain 4: Psychotherapy and Related Theories (30 questions & Answers) 2022 The risk of being overly empathetic as a therapist is commonly characterized by which of the following responses by the client? a. re-enacted attachment with merger and fusion b. re-enacted attachment with understanding and change c. re-enacted attachment with alienation and isolation d. re-enacted attachment with dependency and acting-out The literature on group outcomes has revealed that negative group outcomes have been associated with which of the following qualities in group leaders? a. leaders who initiate discussions of feelings b. leaders with a timid interpersonal style c. leaders who are overly confrontational d. leaders who reveal too much about their personal experience When screening potential female clients for an incest survivor's group, which client would you exclude from selection to the group? A. 36-year-old divorcee who brings a signed consent form to consult with her individual therapist. B. 26-year-oldse her husband is concerned about her lack of interest in sex. C. 50-year-old third grade teacher who last attended a sexual abuse group when flashbacks recurred. D. 18-year-old college freshman who is talking openly about her incest experience for the first time. When working with a family in therapy, you plan to address differentiation of self, triangles, and genograms. What family therapy approach will you be utilizing? A. Strategic family therapy B. Emotionally focused family therapy C. Systemic family therapy D. Structural family therapy In evaluating a 9-year-old girl referred by the school for evaluation of behavioral problems, which of the following reported behaviors raises the greatest concern? A. Difficulty staying on task during school day B. Incomplete and late homework assignments C. Seeming to daydream looking out classroom window D. Chasing boys with provocatively posturing hips on playground As a PMHNP using Family Systems theory, you know that families seek equilibrium in the face of change. This is an example of which concept? A. Homeostasis B. Homeogenesis C. Morphogenesis D. Morphostasis When working with an avoidant patient with a history of trauma, what type of communication techniques are helpful? A. Techniques to increase arousal B. Clarification and close-ended questions C. Reflection and open-ended questions D. Techniques to decrease arousal Which of the following would not be a recommended practice in psychotherapy with patients diagnosed with borderline personality disorder? A. Therapist as a passive listener B. Utilization of concomitant individual and group approaches C. Mutually agreed limit setting D. Establishment of clear roles of patient and therapist A child who is seen by the PMHNP has difficulty feeling safe with his caregiver and is unable to feel comfort. Which theorist's theory would help explain the child's behavior and help guide treatment? A. Bowen B. Bowlby C. Erickson D. Freud When working with adolescents who have poor affect regulation and limited response flexibility, therapists strive to provide a therapeutic frame open enough for changes and consistent enough for stability and safety. Which of the following responses characterizes such a therapeutic frame? A. Detached, neutral, objective B. Empathetic, consistent, strict C. Mindful, reflective, intentional D. Charismatic, engaging, expressive Cognitive behavioral therapy (CBT) is often one of the first-line treatments for anorexia nervosa. One of the primary goals of CBT is to assist the patient in which of the following ways? A. Restore weight above 85% of normal B. Improve interpersonal relationships C. Overcome distorted thinking D. Resolve symptoms of depression An adult female client who has been a full-time homemaker raising four children finds herself sad, regretful, sleeping late into the day, and generally fatigued since her youngest child left for college. What theoretical framework is most appropriate in the PMHNP's appraisal of this situation? A. Mead's role and gender theory B. Erikson's eight stages of psychosocial development C. Family life cycle D. Maslow's hierarchy of needs Johnny, an 8-year-old male, presents with his foster mother for an initial evaluation. When interviewing the foster mother alone, it is confirmed that Johnny was taken out of his home due to sexual abuse by his stepfather. As the treating PMHNP, you know which of the following are evidence-based treatments for Johnny: A. Interpersonal therapy B. Behavioral therapy C. Family therapy D. Trauma focused cognitive behavioral therapy The group leader wants to help the group members work through the feelings of emptiness that all the group members described as one of their reasons for wanting to be part of the group. In an effort to do this, the leader uses interventions that will increase their awareness of choices and responsibility for the consequences of their choice. This approach is congruent with the following theoretical framework: A. Gestalt B. Existentialism C. Psychodynamic D. Bowen Tommy, a 10-year-old boy, is brought to the clinic for a follow-up appointment. During the visit, his mother asks the psychiatric mental health nurse practitioner (PMHNP) if it is normal for Tommy to fixated on rules and is not distressed when things are out of place in his room. Understanding Jean Piaget's stages of development, the PMHNP is aware that Tommy is having difficulty mastering what cognitive developmental stage? a. Stage of formal operations b. Stage of concrete operations c. Sensorimotor d. Stage of preoperational thought A number of theories with biologic, psychological, and social underpinnings have been postulated to explain violence between intimate partners and in the family. Which statement best illustrates the psychosocial theory of intergenerational transmission of violence in explaining why men become batterers? a. Men with traumatic brain injury are prone to seizure disorders, attentional dysfunction, and aggressive behavior. b. Men who witness violence in their family of origin often perpetuate violent behavior in their families as adults. c. Men with alcohol and substance use disorders have disinhibition of social restrains and higher prevalence of violent behavior d. Men with personality disorders including antisocial, borderline, narcissistic, and dependent are prone to violence toward intimate partners. In co-leading a group of elementary children of divorcing parents, a child shared that his parents were arguing over the weekend and calling each other names. What is the most appropriate intervention at this junction? a. Allow the child to finish describing the situation and then ask how he felt about hearing this argument b. Ask the child how he was affected by the incident and ask other children how they handle similar situations c. Ask the other children in the group if they experienced similar situations and how they handled their feelings d. Ask the child how he was affected by the incident and facilitate sharing of similar incidents by other children What is the main differentiating factor between repression and suppression? a. Repression is unconsciousness and suppression in involuntary b. Repression is involuntary and suppression is voluntary c. Repression is voluntary and suppression ins involuntary d. Repression is conscious and suppression is unconscious A patient is followed by the PMHNP psychotherapist and states, "I went to see my primary care provider, and she told me my liver is working harder than it should and she thinks it might be due to my drinking. I don't think I have a drinking problem, but I don't want to have any health problems either. Can you give me the names of where Alcoholics Anonymous groups are located? I told my primary care provider I would try and cut back a bit." What state of readiness to change is this patient in? a. Precontemplation b. Preparation c. Contemplation d. Action Psychotherapy which stresses the importance of recognizing patterns of thoughts and how thoughts influence feelings and behaviors best describes: a. Group therapy b. Supportive therapy c. Family therapy d. Cognitive behavioral therapy During a trauma recovery group a new member has a flashback of her sexual assault. She struggles to regain emotional control. What is the most appropriate intervention strategy at this juncture? a. Use eye contact, ask her to push her feet firmly on the floor, and speak with her in the present so that traumatic feelings can be talked about as memories. b. Monitor the responses of other group members that may have similar flashbacks triggered by the description of this new member's traumatic event. c. Assist her in regaining emotional control by orienting her to the present time and place. d. Encourage the woman to describe the event and associated thoughts and feelings at that time to facilitate desensitization. In working with elderly residents at a nursing home, the goals of reminiscence groups include all of the following except: a. Improve cognitive function b. Reduce wandering c. Facilitate life review d. Maintain self-esteem According to Erikson, the primary task of adolescent psychosocial development is "identity formation" and the associated virtue of "fidelity" to one's nascent self-definition and to one's ideology of "self-in-world." Which of the following behaviors would indicate successful navigation of this developmental milestone? a. Primary identification with "goth" friends despite ostracism by others b. Ability to tolerate differences in others while affirming one's own values and beliefs c. Ability to form a sexually intimate relationship sustained over several years d. Easily identifies with a series of different groups from athletes, rave parties, computer geeks to heavy metal bands. Attachment relationships between infant and mother are considered critical by neurobehavioral researchers because: a. The mother functions as a regulator of the social and emotional environment. b. Attachment to significant relationships facilitates self-regulation of emotions. c. Subtle emotional regulatory interactions can transiently or permanently alter brain activity levels. d. Interaction influences development of the brain's limbic system. The psychiatric mental health nurse practitioner implements a psychoeducation group for parents on the importance of developing secure attachments between the child and parent, and includes information about normal growth and development at specific ages. This is an example of what level of prevention? a. Secondary b. Primary c. Indicated d. Tertiary Understanding normal growth and development, at what age should a child use up to 1,000 words, use three-to four-word sentences, balance on one foot, and play "make-believe"? a. 2 years of age b. 3 years of age c. 4 years of age d. 5 years of age Which of the following brief intervention statements incorporates the key elements of motivational interviewing to reduce risk related to binge drinking at a college campus? a. John, you were fortunate to avoid a car crash after leaving the frat party intoxicated. No one can force you to change your drinking patterns, but I think you can drink in a responsible way and need to limit yourself not to exceed three drinks at a party. Let's discuss some ways to cut back that will work for you. b. Bill, you could have diet from alcohol poisoning with a blood alcohol level that high (2.4 mEq/L). Lucky that your roommate called the paramedics and got you to the Emergency Department. How can I get you to stop the binge drinking? I know you don't want your parents to know about your drinking problem. Let's discuss some ways to cut back that will work for you. c. Mary, you know the risk of STDs from unprotected sex and you know what happens when you drink at frat parties. I can provide free condoms, but the real solution is getting you to cut back on your drinking. Let's discuss some ways to cut back that will work for you. d. Natalie, your drinking and driving scares me. You're lucky that you or your friends in the car weren't injured. I know you've tried to cut back on drinking at the frat parties, but something's not working. You need to stop after two drinks. Let's discuss some different ways to help you do that. Sam, a 6-year-old female, presents with her mother for a new psychiatric evaluation. Her mother reports that Sam is not making friends at school, stays by herself, will not read aloud in class, and requests adult reassurance before making a decision. Which stage of Erik Erikson's Stages of Development is Sam having difficulty developing? a. Trust versus Mistrust b. Industry versus Inferiority c. Identity versus Role Confusion d. Intimacy versus Isolation The PMHNP knows that the ego is a part of the personality and is the logical/rational mind includ8ing defense mechanisms. What might this personality part say? a. "I ought" b. "I should" c. "I evaluate" d. "I want" The following scenario is an example of the defense mechanism sublimation: a. An employee is embarrassed by a boss at work and angrily cuts a driver off on the way home. b. A group therapy client strongly dislikes another member but claims that it is the member who dislikes her. c. A host despises one of the party guests yet greets him warmly and offers him food, beverages, and special attention. d. A mother of a child killed in a drive-by shooting becomes involved in legislative change for gun laws and gun violence. ANCC IQ Domain 5: Ethical and Legal Principles (72 questions and Answers) You have students from local psychiatric nurse practitioner programs follow you in your clinical practice to show them what you do as a Psychiatric Nurse Practitioner. This is called being a: a. Preceptor b. Mentor c. Professor d. Teacher A Psychiatric Mental Health Nurse Practitioner providing psychiatric consultation makes a home visit after the police found an 86-year-old widow wandering the streets at 4AM after emergency call from her daughter. The woman has been living with her daughter for the past ten years, after her husband died. Her daughter is a 66-year-old, single, retired schoolteacher, appears thin, exhausted, and reports little sleep for past two years trying to care for her mother. The daughter reports that her mother becomes increasingly confused every evening and wanders the house unable to sleep at night. The daughter is unable to leave her mother alone for even a short period of time because unsupervised her mother will turn on the gas stove burner, leave it unattended, and she escapes the house at every chance to wander the neighborhood. Her mother is socially pleasant, disoriented to time and place, and does not recognize her daughter. Her mother thinks that the daughter is a housekeeper intent on stealing her money and possessions, which is upsetting to the daughter. What is the most important in determining the level of care needed for this elderly client? A. Appraisal of daughter's health and her ability and willingness to continue caring for mother in home. B. Modification of environment such as bright lights in evening, identification bracelet, and safety lock on stove. C. Trial of low-dose atypical antipsychotic to reduce paranoid ideation, help sleep, and decrease wandering. D. Linkage with home health aide 4 to 6 hours, three times per week, and community resources for respite care. As a psychiatric nurse practitioner, you evaluate your patients for health concerns and beyond, in a holistic manner. your current patient has a blood sugar of 186 and a hemoglobin A1C of 6.7. You refer this patient to an internal medicine practitioner with whom you frequently share patients. You have just violated: A. The DATA Act B. The Adams Act C. No statute D. The Stark law Which of the following is not insured by the Universal Bill of Rights for Mental Health Patients? A. The right to refuse a particular mode of treatment regardless of informed, voluntary, written consent, or situation. B. The right to freedom from restraint or seclusion, other than as a mode of treatment during an emergency situation. C. The right to be given a reasonable explanation of one's general mental and physical condition, the objectives of treatment, and the possible adverse effects of recommended treatment. D. The right to ongoing participation in the planning of mental health services provided in a manner appropriate to a person's capabilities. During an outpatient medication evaluation a depressed client reports persistent anger toward his former boss after losing his job due to arguments and assaultive behavior toward co-workers. The client has been waiting outside the plant in the afternoon, watching for the boss to leave to confront him regarding his terminated employment. What responsibility does the PMHNP have in this situation? A. Contact the client's boss to notify him of potential harm. B. Ask the client to sign a "no harm to others" agreement and document in chart. C. Consult state board of nursing regarding state law requirements on reporting potential harm. D. Advise the client to stop going to the plan and avoid any contact with his former boss. Your patient is a 32-year-old who is diagnosed with major depression-recurrent, insomnia, and chronic lower back pain. The patient has seen you for three medication management sessions over the last two months. During the fourth medication management session, the patient states that money is tight and an appointment is required with the general practice (GP) physician every time a refill of the Hydrocodone (pain medication) is needed. The patient asks you to refill the Hydrocodone to save the cost of another medical visit. you: A. State that you certification is for psychiatric care only and refer him back to his GP. B. State that you are willing to discuss alternative medications to manage his pain. C. State that you can refill this prescription just once. D. State that you would be willing to call his GP to discuss the issues and help him out. You are a nurse practitioner (NP) who performs psychotherapy. your patient is distraught, and you call in the NP who is performing the medication management to sit in and discuss and determine if an emergency medication evaluation is needed. After the discussion of medication changes that will take place, the medication manager NP asks the crying patient if she needs a hug. The patient stands and extends her arms outward. The medication NP gives her a chaste hug and leaves the room. Is this a case of inappropriate touch or battery? A. Yes. This is inappropriate, as you never touch a patient under any circumstances. B. No but, nonverbal communication methods are not sufficient for ensuring agreement to hug a patient. C. Yes. This is a case of battery. The patient did not verbally say that it was okay to touch. D. No. This is a case where there was a question asked, nonverbal consent was given, and the NP gave the patient a hug to help the patient cope. A social worker concerned about a Nigerian refugee who seems to be suffering from severe post-traumatic stress disorder since arriving in U.S. three months ago referred this man to an African American female Psychiatric-Mental Health Nurse Practitioner in the community mental health center. The PMHNP builds rapport by exploring the values, beliefs, life experiences, and accepted behaviors of this young Nigerian man as she seeks to understand the context of his transition to the U.S. before making any diagnosis or proposed intervention. The PMHNP is striving to provide what type of mental health care? a. Alternative care b. Individualized care c. Culturally competent care Culture specific car You are a psychiatric nurse practitioner (NP) and have many patients with comorbid medical and mental health issues. As a part of your continuous quality improvement (CQI) process, you might best serve your practice by: a. Develop a journal group with a general practice colleague to read the latest articles on self-care for advanced practice nurses. b. Subscribe to Medscape and read it daily to see about new medications being developed for personality disorders. c. Developing a peer review process of your charts with a general practice colleague. d. Subscribe to Medscape Psychiatry and read it critically noting studies that apply to your patient's access to care issues. The Nurse Practitioner (NP) attends a local meeting where allocation of resources for healthcare will be discussed. One of the stakeholders has a pamphlet that describes the types of healthcare providers available to help persons in need of health care. The NP notices that Nurse Practitioners are labeled as "Paraprofessionals". In the meeting, the NP asks the organization responsible for the pamphlet why they have identified the Nurse Practitioners as paraprofessionals when a Nurse Practitioner is by law working as 85-90% of the same capacity as a Medical Doctor. The NP goes on the explain what a Nurse Practitioner does in various areas of practice. This is an example of: a. Promoting nurse practitioners in the community b. Public education on the NP scope and standards of practice c. Correcting misinformation that deliberately misleads the public d. Advocating for the role and value of the Nurse Practitioner As a nurse practitioner (NP), you teach your patients that mental illness is no different from any other physical illness that a person suffers. In classes for the community, you emphasize that everybody has a brain and it is another organ in the body. You remind the audience that illness in the brain is no different from illness in the heart, lungs, or liver. This is an example of: a. Reducing Stigma b. Cross education c. Holistic Dynamism d. Integrated health The Veterans Health Administration (VHA) is asking to let its advanced practice registered nurses (APRNs) work independently, even in states that normally require physician oversight. You begin a writing campaign within your state nursing association and American Psychiatric Nurses Association to convince your state's congressman and senators to vote for the bills associated with this legislation. You are: a. Advocating for the total removal of physician supervision of Nurse Practitioners (NP). b. Advocating for the role and value of Nurse Practitioners (NP). c. Advocating for the healthcare of the nation's veterans. d. Advocating for the Nurse Practitioners (NP) to practice to the full extent of their education and training. Which of the following is not associated with increased validity in a clinical research trial? a. Outcome measure b. Sample size c. Confounder d. Randomization A 45-year-old woman with Type I Bipolar Disorder experiencing an acute episode of depression with psychotic features requires a temporary leave of absence from the job as an administrative assistant to a school principal. The patient has signed a release of information for the PMHNP to disclose full mental health records to the principal and the human resources department to facilitate her gradual return to work. The patient fears that she will lose her job due to her psychiatric disorder. What is the most appropriate information the PMHNP can provide in this situation? a. Employers cannot discriminate against individuals with a mental health disorder, which is considered equivalent to a medical disorder by the Mental health Parity Act of 1996. b. Employers cannot discriminate against individuals with a mental health disorder, which is considered a disability covered by the Americans with Disability Act of 1990. c. Employers cannot discriminate against individuals with a mental health disorder, which is protected as a confidential health care information by the Health Insurance Portability and Accountability Act of 1996. d. Employers cannot discriminate against individuals with a mental health disorder, which is a right insured by the Universal Bill of Rights for Mental Health Patients of 1980. You see a patient for a routine medication visit. At. The end of the session, the patient asks questions and the session. Ends. Up 50 minutes in length. You normally charge for the 30-minute appointment, but instead you charge for the 1-hour appointment. The 1-hour appointment includes a full body assessment that you did not perform. This violation is known as: a. Down-coding b. Super-coding c. Over-coding d. Up-coding A primary care nurse practitioner consults the PMHNP for. Guidance on medication management for a family member she is prescribing medication for bipolar I disorder, because the family member does not have health insurance and cannot afford to see a mental health provider. What would be the most appropriate response by the PMHNP? a. Recommend that she not manage a family member's medication and recommend free clinic. b. Determine acuity and phase of bipolar I disorder to advise on appropriate generic medications c. Recommend free clinic and report nurse practitioner to state board of nursing for practice violation. d. Offer to see the patient pro bono and manage medications. As a psychiatric nurse practitioner, you frequently are asked to "case manage" your patients for more than psychiatric issues. Your 68-year-old patient, with Medicare is the primary insurance, states that varicose veins are an issue and that treatment is needed. You know that your sister is the best vein surgeon in your city and refer the patient too her. You have just violated: a. The Stark Law b. The Harter Act c. The Adams Act d. The DATA Act The Mental Health Parity and Addiction Equity Act of 2008 (MHPAEA) insures which of the following? a. Mental health and substance use disorder benefits must be available through both in-network-providers and out-of-network providers by a group health plan. b. Group health plans may obtain nan exemption if they can demonstrate expected cost increase resulting from implementation of the parity provisions greater than a 5% increase in the cost under existing plan. c. Small employers who insure 50 or fewer covered lives are exempt from the provisions of this MHPPAEA law. d. Annual or lifetime dollar limits on mental health and substance use disorder benefits are no lower than any such dollar limits for medical and surgical benefits offered by a group health plan. You are a Psychiatric Nurse Practitioner (NP) working in a hospital health system. The hospital administration has implemented a productivity system where all the Psychiatrists and NPs in the department of psychiatry will now have to see four patients per hour. You have informed administration that this is not a good system because many patients are in fragile mental and physical health, require case management, and need extensive care during their medication management sessions to achieve good management of their symptoms and recovery. The hospital administration has reinformed that the productivity system will stay in place and is mandatory. If it is not met, persons not meeting the quota will be penalized. You inform the administration that you cannot work under these conditions. Administration informs you that you are abandoning your patients. You: a. Make the decision to leave your position. Tell your patients the hospital is treating them poorly. b. Make the decision to stay, telling your patients the hospital is treating them poorly. c. Make the decision to leave your position, as you cannot support a system that disregards the needs of the patient. d. Make the decision to stay, as you cannot abandon your patients. The Nurse Practitioner (NP) has a Law Bill in the State Legislature passed so that funds will be released to be used for housing the severe and persistently mentally ill. The NP contacts eight persons and ask them to telephone the State Senator who heads the Health and Human Services committee. She asks them to tell the State Senator to vote to approve the bill. The NP is acting as an: a. Advocate b. Policy maker c. Stigma Reduction Agent d. Care Provider Your patient is receiving Medicare and Social Security Disability payments. This money helps her to pay for food, clothing, shelter, and medications. She recently began to work part-time. Her work has greatly improved her mood and self-perception. She has taken on more work. The additional income has caused her to lose her disability and Medicare funding. She is concerned that she will not be able to get her medication, pay for housing, and some of the other things she was able to afford before. How do you counsel her? a. Discuss with her your assessment that her mood has improved since working and self-supporting. Assist her to find programs that will help her to continue. b. Advise her that her counselor can help her with financial issues-programs, services, and resources. c. Advise her to maintain her Medicare and Social Security benefits because she will always have her food, clothing, shelter, and medication support. d. Discuss with her the consequences of taking on "too much" work ad losing her disability benefits. The Nurse Practitioner (NP) is seeing a patient for both psychiatric and chemical dependency illnesses. The patient wants the NP to send medical records to his General Practitioner. The NP tells the patient that because of Federal Law 42 CFR he must: a. Sign two separate releases of information due to the special protected nature of chemical dependency information. b. Sign on combined release that designated that he is being treated for both chemical dependency and psychiatry illnesses. c. Sign two separate releases due to the special protected nature of psychiatric information. d. Sign one combined releases that does not mention his chemical dependency illness due to its special protected nature. Psychiatric-Mental Health Nurse Practitioners seeking admitting privileges at a community hospital with inpatient psychiatric services need to influence policymakers at which of the following levels? a. State Legislature b. County board of health c. State board of nursing d. Local hospital Your new 30-year-old patient with Generalized Anxiety Disorder has been on Alprazolam (Xanax) 0.5 mg 1 tab PO BID prn anxiety for the past 7 years after trials of multiple SSRI's. She uses this medication about two times per week for "extreme" anxiety. Your personal bias as a careful psychiatric nurse practitioner is that benzodiazepines should only be used with other non-addictive anti-anxiety medications and internally generated copping skills to manage anxiety. In discussing the case with the patient, you recognize that she has been carefully using the Alprazolam, has tried other methods o reduce anxiety, and is working with a therapist. What will your plan of care be? a. The patient is coping well with a limited amount of Alprazolam. Reinforce safe use of benzodiazepines. Continue the medication and encourage use of more internal coping mechanisms. b. The patient may do better with increased therapy. Consult the therapist and wean her off the Alprazolam. c. The patient is using Alprazolam too often. Reinforce safe use of benzodiazepines. Help her to wean off the medication and use more internal coping mechanisms. d. The patient is coping well with the limited amount of Alprazolam, but may do better with Atenolol (Tenormin) 25 mg po QHS. Change the medication prescribed. You notice that you have begun to care less about your patient's needs. You are tired, unhappy and unsatisfied inn your job. You talk to your office confidant, another psychiatric nurse practitioner about your feelings and concerns about how this might impact your practice. Together, you discuss your need t: a. Reduce your present workload to get more rest and practice better living, eating, exercise, and living skills. b. Avoid those things about your present employment that upset you and get involved in a yoga class. c. Leave your present employment and find a different career in nursing that fulfills you better. d. Leave your present employment in nursing and fid a different career path in something that fulfills you better. To demonstrate her support for social justice, within her role as a Psychiatric Nurse Practitioner caring for the person who is homeless with mental illness, the psychiatric nurse practitioner (NP): a. Volunteers her clinical services at the psychiatric clinic, run by a local mission project. b. Teaches Sunday School. c. Volunteers to serve food at the local Home for Girls. d. Volunteers to walk at the March of Dimes walk each year. Per the Health Insurance Portability and Accountability Act (HIPPA), a Nurse practitioner (NP) who is in an independent, single practitioner practice is considered a "Covered Entity" because: a. The NP transmits protected health information in electronic forms. b. May accidently release protected health information. c. Writes information and generates protected health information. d. Maintains charts with the patient information that contains protected health information. Your patient is a 37-year-old male who you have diagnosed with major depression. He was referred to you by a Licensed Alcohol and Drug Abuse Counselor (LADAC) who has diagnosed him with Opioid Use Disorder-Severe. After your evaluation of the patient, you concur with the LADAC's diagnosis and treatment with an antidepressant and inpatient chemical dependency as his best chance for recovery. You discuss treatment with the patient and tell him the rationale for your treatment choice. You discuss his failure to maintain abstinence, his health issues, the loss of friends and family, and other issues that surround using. He refuses the treatment, citing that he is able to care for himself and will continue to try on his own to maintain abstinence. He states that he cannot go through inpatient treatment but will try antidepressant medication and continue seeing his therapist on an outpatient basis. You: a. Continue to discuss the benefits of inpatient treatment and the health needs of the patient until he accepts the recommendation. b. Dismiss the patient from your service. c. Contact the LADAC counselor to manage the patient to place him in protective custody. d. Accept his right to determine his own treatment path. A newly graduated Nurse Practitioner (NP) wants to open a private psychiatric practice in her hometown. She wants to find out about the laws that cover her responsibility and legal liability that this practice will encounter for her as an independent psychiatric nurse practitioner. Her best source of information is: a. Local Psychiatric Nurse Practitioners in practice. b. The American Academy of Nurse Practitioners. c. The State Statutes on Nursing or Advanced Nursing practice. d. Her City Statutes. A colleague emails on the company computer system regarding complaints about another coworker. In the message, the colleague makes several disparaging remarks about the other coworker and the difficulty of working with her. You reply: a. She is just not doing things the way we were taught. She can't help it if she is not us, but she could try harder! b. Really, I cannot believe that you get so upset over that little stuff. Help her understand where she is making a mistake and teach her how to do things correctly. c. I understand your feelings, just put up with it. You are going to another shift in one month and will leave the problem behind. d. Did you talk with her about the issue you have with her? Saying things about your co-worker will not resolve the problem or make it better. The core values that underline advanced practice nursing and culturally competent care include: a. Care, advocacy, respect b. Autonomy, respect, collaboration c. Respect, advocacy, partnership d. Care, respect, collaboration The Nurse practitioner (NP) is seeing a 72-year-old male for depression management. Upon assessment, the patient is alert, oriented, and non-suicidal or homicidal. The patient is able to manage his activities of daily living. The NP witnesses the man's wife verbally abusing him on several occasions. The NP has verbal reports from staff nurses and their nursing notes that the man has come into the unit with bruising and contusions inconsistent with his affluent lifestyle and physical health condition. When the NP speaks to the man about the physical and verbal abuse, he tells her that it is occurring. He also tells her that he does not want to do anything about the situation. He tells her specifically that he does not want her to contact any authorities about the situation. a. She is required to contact Adult protective Services or similar services in the local community to investigate the senior citizen's safety. b. Take pictures of the brises and contusions and save them to the patient's record. c. Arrange to remove the patient to a safer environment. d. The NP must do as the patient requests as long as he is competent. As a psychiatric mental health nurse practitioner who is working in a larger health system and understands models of change, which of the following processes are typically included in these models? a. Engaging stakeholders, addressing organizational barriers, and evaluating the impact of the practice change. b. Engage the unit-level manager, assess unit barriers, and obtain funding to implement the change. c. Conduct a literature search, write the proposal, and obtain funding for the change. d. Form a committee, identify the need, and evaluate staff satisfaction. 7. A frail 76-year-old woman with Alzheimer's type dementia in a skilled nursing facility has been increasingly agitated, combative, and has struck out at other patients in the dining room and at staff attempting to care for her. What would be the most appropriate initial intervention by the consulting PMHNP who receives a call from the nursing staff requesting medication and restraint orders? a. Order close observation by staff in quiet area secluded from other patients until you can evaluate in person. b. Order low-dose atypical antipsychotic and soft vest restraint until you can evaluate in person. c. Order short-acting benzodiazepine and close observation until you can evaluate in person. d. Order soft wrist and vest restraints until you can evaluate in person. During a medication follow-up appointment at student mental health clinic, a 20-year-old college sophomore with depression and history of binge drinking disclosed that she has been struggling with recurrent intrusive thoughts to overdose. She recently broke up with her boyfriend when she learned that he was sleeping with her closest girlfriend. She admits to taking a handful of her sertraline prescribed by the PMHNP last weekend and was frustrated that it only made her sleep for a day. She's angry and ambivalent about the future, unwilling to make any promise not to harm herself, and has made veiled threats to "pay back both of them for cheating" (referring to her ex-boyfriend and girlfriend). What is the most appropriate intervention at this juncture? a. Ask the student to promise that she won't harm herself or anyone else and schedule next day counseling visit. b. Ask the student if she will agree to voluntary admission and arrange for involuntary admission if necessary. c. Counsel the student that she would regret hurting herself and can move beyond this broken relationship. d. Negotiate with the student to touch base daily by phone between counseling visits to avoid hospitalization. During a session, your patient asks to be able to contact you via your Facebook page on the internet. You reply: a. I cannot do that because Facebook does not protect your Private Health Information. b. I would be happy to use Facebook but you will have to use a private message so that you can be anonymous. c. I cannot do that because I have a concealed Facebook identity; and I cannot reveal that to my patients. d. I would be happy to communicate with you. It makes it easier for me to reach you this way. The Health Insurance Portability and Accountability Act of 1996 (HIPPA) is a federal law intended to protect employed Americans' privacy, confidentiality, and access to health care through health insurance coverage. Knowledge of major HIPPA provisions enables the PMHNP to advise patients regarding health care coverage. Which of the following statements regarding HIPPA is true? a. HIPPA replaces the State as the primary regulator of health insurance. b. HIPPA increases ability for workers changing jobs to get health coverage. c. HIPPA guarantees health coverage for all workers. d. HIPPA eliminates all use of pre-existing condition exclusions. Your patient wants to communicate with you through email. You have a secure email account that is encrypted and will protect health information as required by Health Insurance Portability and Accountability Act (HIPPA). Your patient does not have an encrypted email account. Your patient does not have an encrypted email account. Your rely is: a. I can encrypt our email communications too be sure that your protected health information (PHI) is unavailable to others and protect you. b. I do not like to communicate through email I would refer to talk to you in person or over the telephone only. c. I have no problems with you communicating with me through my personal email. d. In order to communicate back and forth, you will have to find a way to encrypt your protected health information (PHI). As an individual advanced practitioner, you cannot afford to have a standalone Electronic Health Record (EHR) system to keep all of your patient's health information on a computer-based system. How could you problem-solve this dilemma? a. Process your patient records manually, in your current fashion. You are keeping a full record according to HIPPA. b. Process your notes and patient records on a laptop that you are able to transport between your office and home office. You always have the records available if needed. c. Process your notes and patient records in a secure electronic system with a template for patient information and interactions, supporting quick access within your network. d. Process your notes manually and keep them in a locked file. The records are available for copy and distribution at any time. Your patient is a 40-year-old male who has suffered from depression since he was age 21. He has tried numerous antidepressant medications and combinations of medications to no avail. There is a response but no recovery for this patient. He is currently taking Duloxetine 120 mg 1 po QAM with a response. You prescribe a new medication to help with resistant depression, Brexpiprasole (Rexulti) 0.5 mg 1 po QAM. He tries samples of the medication for 1 month and he experiences recovery from depression. When he takes the prescription too the pharmacy, he finds that his insurance will not cover the cost of the medication sufficiently. The patient tells you that he just cannot afford the medication and wants stop it. What do you do? a. Work with the pharmacy representative to get as many samples as possible. Give them all to the patient so that he has a supply of the Brexpiprasole. b. Refer the patient to a psychiatrist because you cannot help him. c. Stress the need for recovery and push for him to make the financial sacrifices to afford the Brexpiprasole. d. The patient is not in danger of self-harm and has the right to decline his own health. Continue to research other medications that will help him. Your work as a nurse practitioner (NP) makes you aware that there are many people who cannot afford to get services for their mental health needs. As part of your concern for social justice, you: a. Care for a number of patients pro bono each year. b. Act as a delegate at your local nursing convention. c. Support a local candidate for office who is "progressive" about health care. d. Act as a health screener at your child's kindergarten. A mobile crisis team responds to an emergency call by a residential care home (RCH) manager. A 65-year-old resident had become increasingly agitated, insisted that the RCH cook was poisoning her food, and verbally threatened the cook. The RCH manager had found several days of the resident's medications hidden in her dresser drawer. Which is the most appropriate intervention by the mobile crisis team? a. Admission to a nursing home. b. Admission to the gero-psychiatric unit at the community hospital. c. Admission to the state psychiatric hospital. Admission to the crisis stabilization unit You notice that you have begun to care less about your patient's needs. You are tired, unhappy, and unsatisfied with your job. You talk to your office confidant, another psychiatric nurse practitioner about feelings of "compassion fatigue." He indicates that while he can understand your concerns based on your self-assessment, but he does not believe you have compassion fatigue because he has not observed you: a. Staring off into the distance, suddenly crying for no reason, rapid changes in mood, impulsive spending, valuing and devaluing people. b. Constantly tired, wanting to do more of the paperwork and less of the patient work, wanting to stay longer and chat in the break room rather than be in your office alone. c. Heavily criticizing others during meetings, sitting in your office alone all the time, dressed in the same outfit all the time, and in denial about your problems. d. Hyperactive but having little productive work to show for it, poor concentration, not listening to others despite being present, and constantly forgetting things needed for daily work. You frequently use your iPhone/smart phone to connect with your office, the pharmacy, and patients. These multifunction devices assist you in your practice. What needs to happen to the protected/private health information on your phone after you conclude a healthcare interaction? a. You document an abbreviated note of the phone conversation. b. You document that you had a telephone interaction only. c. You document it briefly, only if it is important. d. You document it fully in the patient record as a patient interaction. When counseling a woman with children who is living in a violent domestic relationship, the PMHNP needs to advise that: a. Restraining orders can be helpful in preventing further violence. b. Anxiety, depression, dissociation, and PTSD symptoms are common in survivors. c. Shelters routinely provide care for both mothers and their children. d. Written material about shelters found by the perpetrator may trigger further violence. You are treating a new patient. He indicates that he is a Muslim and follows all of the principles of that religion. He is suffering from depression. What special considerations should you take to help him in his treatment. a. Refer him to a male practitioner for medications and continue to research medications that do not contain any fish or animal fats. b. Refer him to a male therapist and continue to research medications that do not contain vanilla products. c. Refer him to a male practitioner for medications and explain that patient's need to use medications that contain no gelatin products. d. Refer him to a male therapist and continue to research medications that do not contain blood or horse byproducts. Which of the following would not be included in a progress note? a. Incident report on medication error. b. Patient's self-rating on target symptoms. c. Risk/benefit analysis for change in medication. d. Standardized outcome measures The guidelines that a group, community, or organization develop regarding its goals, direction, and priorities, and the way it implements its resources to attempt to reach those goals is the definition of: a. Law b. Policy c. Politics d. Precedent A 37-year-old male with schizophrenia, paranoid type, lives in supervised housing and is followed by the Assertive Community Treatment Team through a Community Mental Health Center (CMHC) where the PMHNP notes that he has refused to see any primary care provider (PCP) during the past several years despite efforts by case managers to accompany him to medical visits. He has gained 30 pounds in the past three years on clozapine since his last inpatient hospitalization. His WBC and ANC counts remain within normal limits, but he has elevated cholesterol, lipids, and A1C levels. What is the most appropriate intervention to facilitate care for this patient? a. Advise the patient that he needs to see a PCP to treat his elevated cholesterol and blood sugar levels and ask his preference for referral to one of the CMHC's affiliated PCPs. b. Advise the patient that he needs to see a PCP to treat his elevated cholesterol and blood sugar levels and ask what has prevented him from seeing a PCP when referred in past. c. Explain the health risks of his elevated cholesterol and blood sugar levels and that he may eventually have to be hospitalized for medical treatment because this is a serious problem. d. Explain the health risks of his elevated cholesterol and blood sugar levels and separately ask the patient and case manager what has prevented him from seeing PCP when referred in the past. The nurse practitioner (NP) sees an opportunity to help here patient who suffers from anxiety and hypertension by utilizing a medication that can treat both issues. She talks to her patient about using Atenolol 25 mg ½ tab po QHS to manage his hypertension and chronic anxiety. To promote both the patient's continued health and collaboration, the NP: a. Telephones the patient's primary care provider to discuss the use of the Atenolol before starting the medication. b. Faxes a copy of her car notes to the Primary Care Provider (PCP) to inform the PCP of her plan of care. c. Telephones the Primary Care Provider's office and leaves a message about the change in medication. d. Tells the patient to let his Primary Care Provider know about the change in medication. . Which of the following best depicts the PMHNP in a grassroots mental health advocacy role? a. Meeting state legislators during RN lobbying day organized by the state nurses' association. b. Attending a town hall meeting to seek timely police response to rising domestic violence crisis calls. c. Working on the campaign of an individual running for the state senate. d. Assisting with depression screening at a health fair in a local shopping mall. Outcomes measurement is the collecting and reporting of data about the effect of an intervention. Purposes of outcome measurement include all of the following except: a. To profile practice patterns of providers. b. To observe errors in data reporting of an intervention. c. To suggest changes in treatment. d. To analyze the effectiveness of an intervention. A Nurse Practitioner decides to go back to school to gain further knowledge about the neurological basis of psychiatric illnesses. She is not doing this to maintain her licensure but to enhance her own knowledge. This is an example of: a. Continuing Education Credits b. Nursing Informatics c. Nursing Research d. Professional Development A 68-year-old retired African-American widower who served for 30 years as an Army officer was recently diagnosed with terminal lung cancer. He made plans to die at home with hospice care. He was hospitalized for a broken hip and succumbed to complications in the hospital. Despite his request to be addressed as "Mr. Baxter" the hospital staff persisted in calling him by his first name, "John." Which principle of culturally competent care does this violate? a. Respect b. Autonomy c. Advocacy d. Collaboration 43-year-old male patient presents to the psychiatric mental health nurse practitioner (PMHNP) for a medication follow-up appointment. As part of his follow-up visit, his blood pressure reading is 156/102 millimeters of mercury. The patient states he has been having headaches for the last few weeks since he ran out of his blood pressure medication. The PMHNP's best action nis which of the following? a. Contact the patient's primary care provider, explain the patient's symptoms, and request his medication be refilled and he be given a follow-up appointment b. Call a local family nurse practitioner and request that she refill this patient's medication c. Refer the patient to the emergency department for an evaluation d. Call the pharmacy to confirm which antihypertensive the patient is taking and write a 30-day supply of this medication The initial advanced practice role for the Psychiatric Mental Health nurses was the Clinical Nurse Specialist (CNS) that worked as a therapist and educator. Later, the PMHNP role was developed to serve the need of a primary care psychiatric provider that could perform evaluations and medication management services in hospitals and other venues. As of 2015, the Consensus Model for Advanced Practice Registered Nurse (APRN) practice regulation (i.e., licensure, accreditation, certification, and education-LACE) for the Psychiatric APRN role is approved for a: a. Nurse Practitioner that treats the psychiatric mental health population from birth through geriatrics. b. Nurse Practitioner that treats the psychiatric mental health population only. c. Nurse Practitioner that treats the psychiatric mental health population in the hospital setting only. d. Nurse Practitioner that treats the adult psychiatric mental health population only, under the supervision of a physician. When a clinical trial fails to reject the null hypothesis (i.e., when p-value > 0.05), the investigator may have missed a true intervention effect. One should consider the following to avoid a Type II error: a. Power b. Sample bias c. Alpha level d. Effect size You are in a meeting of your state nurse practitioner legislative group. The group's director has asked you to present information on a law bill (LB) that is being presented for public hearing in the state legislature. The information covers how best to approach securing support from the state's Health and Human Services Committee (state senators and congressmen). As you begin to present the information, two of your colleagues on the committee begin to talk over you about their ideas and will not quiet down. The group director does not intervene to settle the meeting. You decide to: a. Talk over the colleagues. Ask them to allow you to finish before they make comments. Take questions at the end of the presentation. b. Clear your voice and ask for attention. When that does not work, stop speaking until the situation resolves. If it does not resolve, sit down quietly until the behavior stops. c. Confront the speakers for their ill behavior and ask them to be quiet until you have finished your presentation. d. Ignore the speakers and ask the group director to settle the room so that you can finish your presentation. The Nurse Practitioner (NP) is working with a patient secondary to an injury that he sustained while working on the job. The NP is managing both the medications and psychotherapy portions of this patient's care. The patient is involved in workman's compensation regarding his injury. The patient's employer sends the NP a release of information form requesting the patient's complete medical record. The NP: a. Talks to the patient before sending any records to the employer. b. Sends the complete record but withholds the psychotherapy notes. c. Sends the complete record including the psychotherapy notes. d. Declines to send the records due to the workman's compensation case. As a Psychiatric nurse practitioner (NP), you accept an RN student who is studying to become a psychiatric NP. You ask the student to perform an initial psychiatric evaluation on a new patient while you observe. As the preceptor, you are responsible for: a. Satisfaction of the patient with the outcome of the evaluation. b. The patient feeling that he or she is being judged for "incarceration" in a chemical dependency treatment program. c. Gaining permission from the patient for the student to perform the evaluation and to ensure the quality of the evaluation. d. Gaining permission from the patient for the student to perform the evaluation. The ability of the student to perform the evaluation is their professional responsibility. Which study design is appropriate in determining the difference in effectiveness of one medication compared to another medication in the treatment of bipolar disorder? a. Cross-sectional design b. Cohort design c. Randomized controlled trial d. Case-controlled trial Which of the following is not the responsibility of Institutional Review Boards (IRB) in the research process? a. Ensure that adverse events are reported and risks/benefits are re-evaluated. b. Ensure that informed consent is obtained and documented. c. Ensure that animal studies are conducted using humane methods. d. Ensure that risks to participants are minimized. The Nurse Practitioner (NP) requires all patients to have a set of laboratory tests done each year to check changes in values as a safety measure. The patient states that these labs were done at his Internal Medicine Physician's office within the last month. The NP states that she will accept those labs as long as they cover all of the labs that she requires for the yearly check. The patient then verbally gives the NPP his Internal Medicine Physician's office number to call and requests that she get the lab results. The NP then: a. Mails a formal request to the Internal Medicine Office and physician requesting the laboratory results. b. Faxes a formal request to the Internal Medicine Office and physician requesting the laboratory results. c. Get a signed release of information for the patient for the Internal Medicine Physician for the laboratory results. d. Calls the internal Medicine office and verbally requests that they fax over the laboratory results. In a therapy session, a five-year-old patient shows the Nurse Practitioner (NP) bruises on her back and legs. The child discloses that her mother's boyfriend hits her and tells her to "shut up". The NP is required to: a. Contact the mother to investigate the situation further. b. Take pictures of the bruises and save them in the child's file. c. Contact Child Protective Services or similar services in the local community to investigate the child's safety. d. Arrange to remove the child from the dangerous situation to protect her. The PMHNP joined the local National Alliance on Mental Illness Board. Serving as a board member to advocate for access to care and parity of services is an example of which standard of professional performance? a. Collegiality b. Leadership c. Practice d. Consultation When interpreting confidence intervals around relative risk reduction, a wider confidence interval reflects that: a. The estimate of effect is more precise b. The estimate of effect is less precise c. There is not statistically significant difference in effect d. There is a statistically significant difference in effect You are considering using text message technology with your patients as a way to communicate quickly in our fast-paced world. What considerations are necessary when texting? a. Text messaging is safe because of advanced security platforms for protecting protected health information (PHI) in smart devices. b. Text messaging is becoming the accepted standard of communication since the availability of means for retrieving protected health information (PHI) from any device. c. The use of text message technology is not recommended by regulatory agencies. d. Text messaging using a secure text messaging platform within specific Joint Commission guidelines could be used for orders only. A PMHNP practices in a state requiring a collaborating psychiatrist. The PMHNP reviews all new and complex cases with the psychiatrist on a monthly basis. A young adult patient with recurrent depression has been under the care of the PMHNP for 3 months when she commits suicide. The family sues for wrongful death. Who is legally responsible for this patient's care in a court of law? a. PMHNP b. Psychiatrist c. Primary responsibility of PMHNP and secondary responsibility of psychiatrist Equal responsibility of PMHNP and psychiatrist To advocate for the skills and value of the psychiatric nurse practitioner (NP), you: a. Arrange for you and other psychiatric nurses to participate in a blood drive for a local psychiatric hospital. b. Arrange for your office to participate in a walk for Alzheimer's Disease. c. Arrange for a local TV station to interview you about what Psychiatric NPs can do for the community. d. Arrange a meet and greet with other nurses at a drug luncheon promoting a psychiatric medication. Your mother calls you at the office and asks you to call in a prescription for her. The medication is for her sleeping pills, Eszopiclone (Lunesta) 3 mg 1 po QHS #30. You: a. Call the script into her pharmacy with three additional refills, just like any other patient. b. Tell her that you will only do it "just this once" for her and then call the script into her pharmacy. c. Call your mother's provider and ask the provider to call your mother's script in for her. d. Tell her to call the provider who normally prescribes that medication for her. In counseling a 23-year-old, married Hispanic mother who brought her 4-year-old son to the clinic for "mal de ojo" with symptoms of fitful sleep, diarrhea, vomiting, and fever, the PMHNP: a. Educates about importance of maintaining fluid and electrolyte imbalance. b. Respect the mother's understanding of the child's illness. c. Identifies what steps the mother has already tried in caring for child's symptoms. d. Explains that the symptoms are most likely caused by viral infection. The nurse practitioner (NP) is on an airplane and the flight attendant asks if there is a medical professional on the flight. The NP agrees to help. The NP is presented with a young woman who is suffering from a severe panic attack. The woman is nauseated and lying on the floor of the plane. The woman has trouble communicating much more than she is upset, afraid, and her husband is a few airplane stops away at an air force base where she is headed. The woman's pulse if 89 and regular and her respirations are 18 and regular. The woman occasionally pants and vomits some pale brown liquid. You provide information to the flight medical doctor on call. At the destination, the flight attendant offers to give you some Alprazolam 0.5 mg to give to the patient. You: a. Give her the Alprazolam to help her calm down until you get to the destination. Then, she can go to the hospital. b. Do not give the medication. Ask her if she would like to take the medication. If she agrees, allow the flight attendant to give it to her. c. Do not give the medication. Instead, wait until the plane lands and a full evaluation can be completed at the local hospital. d. Give her the Alprazolam so that she can calm down and go on to her next flight. [Show More]

Last updated: 1 year ago

Preview 1 out of 81 pages

Reviews( 0 )

$35.00

Add to cart

Instant download

Can't find what you want? Try our AI powered Search

OR

GET ASSIGNMENT HELP
118
0

Document information


Connected school, study & course


About the document


Uploaded On

May 23, 2022

Number of pages

81

Written in

Seller


seller-icon
Courses_Exams

Member since 3 years

71 Documents Sold


Additional information

This document has been written for:

Uploaded

May 23, 2022

Downloads

 0

Views

 118

Document Keyword Tags

Recommended For You


$35.00
What is Browsegrades

In Browsegrades, a student can earn by offering help to other student. Students can help other students with materials by upploading their notes and earn money.

We are here to help

We're available through e-mail, Twitter, Facebook, and live chat.
 FAQ
 Questions? Leave a message!

Follow us on
 Twitter

Copyright © Browsegrades · High quality services·